Peds exam 4: chapters 15, 17, 18, 19, 28, 29

Pataasin ang iyong marka sa homework at exams ngayon gamit ang Quizwiz!

31. The nurse is implementing care for a school-age child admitted to the pediatric intensive care in diabetic ketoacidosis (DKA). Which prescribed intervention should the nurse implement first? a. Begin 0.9% saline solution intravenously as prescribed. b. Administer regular insulin intravenously as prescribed. c. Place child on a cardiac monitor. d. Place child on a pulse oximetry monitor.

ANS: A All patients with DKA experience dehydration (10% of total body weight in severe ketoacidosis) because of the osmotic diuresis, accompanied by depletion of electrolytes, sodium, potassium, chloride, phosphate, and magnesium. The initial hydrating solution is 0.9% saline solution. Insulin therapy should be started after the initial rehydration bolus because serum glucose levels fall rapidly after volume expansion. The child should be placed on the cardiac and pulse oximetry monitor after the rehydrating solution has been initiated.

15. A school nurse is performing hearing screening on school children. The nurse recognizes that the most common type of hearing loss resulting from interference of transmission of sound to the middle ear is characteristic of which type of hearing loss? a. Conductive b. Sensorineural c. Mixed conductive-sensorineural d. Central auditory imperceptive

ANS: A Conductive or middle-ear hearing loss is the most common type. It results from interference of transmission of sound to the middle ear, most often from recurrent otitis media. Sensorineural, mixed conductive-sensorineural, and central auditory imperceptive are less common types of hearing loss.

36. Which of the following types of seizures may be difficult to detect? a. Absence b. Generalized c. Simple partial d. Complex partial

ANS: A Absence seizures may go unrecognized because little change occurs in the child's behavior during the seizure. Generalized, simple partial, and complex partial seizures all have clinical manifestations that are observable.

30. A mother calls the school nurse saying that her daughter has developed a school phobia. She has been out of school 3 days. The nurse's recommendations should include which intervention? a. Immediately return child to school. b. Explain to child that this is the last day she can stay home. c. Determine cause of phobia before returning child to school. d. Seek professional counseling before forcing child to return to school.

ANS: A The primary goal is to return the child to school. Parents must be convinced gently, but firmly, that immediate return is essential and that it is their responsibility to insist on school attendance. The longer the child is permitted to stay out of school, the more difficult it will be for the child to reenter. Trying to find the cause of phobia will only delay the return to school and inhibit the child's ability to cope. Professional counseling is recommended if the problem persists, but the child's return to school should not wait for the counseling.

37. An important nursing intervention when caring for a child who is experiencing a seizure would be to: a. describe and record the seizure activity observed. b. restrain the child when seizure occurs to prevent bodily harm. c. place a tongue blade between the teeth if they become clenched. d. suction the child during a seizure to prevent aspiration.

ANS: A When a child is having a seizure, the priority nursing care is observation of the child and seizure. The nurse then describes and records the seizure activity. The child should not be restrained, and nothing should be placed in the child's mouth. This may cause injury. To prevent aspiration, if possible, the child should be placed on the side, facilitating drainage.

32. Which is an appropriate nursing intervention when providing comfort and support for a child when death is imminent? a. Limit care to essentials. b. Avoid playing music near child. c. Explain to child need for constant measurement of vital signs. d. Whisper to child instead of using normal voice.

ANS: A When death is imminent, care should be limited to interventions for palliative care.

3. Which side effects should the nurse monitor when a child is taking an antipsychotic medication? (Select all that apply.) a. Extrapyramidal effects b. Hypertension c. Bradycardia d. Dizziness e. Seizures

ANS: A, D, E Common side effects of antipsychotic medications include dizziness, drowsiness, tachycardia, hypotension, and extrapyramidal effects, such as abnormal movements and seizures.

4. A nurse is instructing a nursing assistant on techniques to facilitate lip reading with a hearing-impaired child who lip reads. Which techniques should the nurse include? (Select all that apply.) a. Speak at eye level. b. Stand at a distance from the child. c. Speak words in a loud tone. d. Use facial expressions while speaking. e. Keep sentences short.

ANS: A, D, E To facilitate lip reading for a hearing-impaired child who can lip read, the speaker should be at eye level, facing the child directly or at a 45-degree angle. Facial expressions should be used to assist in conveying messages, and the sentences should be kept short. The speaker should stand close to the child, not at a distance, and using a loud tone while speaking will not facilitate lip reading.

1. Nursing care of a child diagnosed with a syndrome of inappropriate ADH should include which of the following? (Select all that apply.) a. Weigh daily b. Encourage fluids c. Turn frequently d. Maintain nothing by mouth (NPO) e. Restrict fluids

ANS: A, E Increased secretion of ADH causes the kidney to reabsorb water, which increases fluid volume and decreases serum osmolarity with a progressive reduction in sodium concentration. The immediate management of the child is to restrict fluids. The child should also be weighed at the same time each day. Encouraging fluids will worsen the child's condition. Turning frequently is not an appropriate intervention unless the child is unresponsive. Fluids, not food, should be restricted.

3. An infant with hydrocephalus is hospitalized for surgical placement of a ventriculoperitoneal shunt. Which interventions should be included in the child's postoperative care? (Select all that apply.) a. Observe closely for signs of infection. b. Pump the shunt reservoir to maintain patency. c. Administer sedation to decrease irritability. d. Maintain Trendelenburg position to decrease pressure on the shunt. e. Maintain an accurate record of intake and output. f. Monitor for abdominal distention.

ANS: A, E, F Infection is a major complication of ventriculoperitoneal shunts. Observation for signs of infection is a priority nursing intervention. Intake and output should be measured carefully. Abdominal distention could be a sign of peritonitis or a postoperative ileus. Pumping of the shunt may cause obstruction or other problems and should not be performed unless indicated by the neurosurgeon. Pain management rather than sedation should be the goal of therapy. The child is kept flat to avoid too rapid a reduction of intracranial fluid.

33. Which is descriptive of central nervous system stimulants? a. They produce strong physical dependence. b. They can result in strong psychological dependence. c. Withdrawal symptoms are life threatening. d. Acute intoxication can lead to coma.

ANS: B Central nervous system stimulants such as amphetamines and cocaine produce a strong psychological dependence. This class of drugs does not produce strong physical dependence and can be withdrawn without much danger. Acute intoxication leads to violent, aggressive behavior or psychotic episodes characterized by paranoia, uncontrollable agitation, and restlessness.

14. A school nurse is performing hearing screening on school children. The nurse recognizes that distortion of sound and problems in discrimination are characteristic of which type of hearing loss? a. Conductive b. Sensorineural c. Mixed conductive-sensorineural d. Central auditory imperceptive

ANS: B Sensorineural hearing loss, also known as perceptive or nerve deafness, involves damage to the inner ear structures or the auditory nerve. It results in the distortion of sounds and problems in discrimination. Conductive hearing loss involves mainly interference with loudness of sound. Mixed conductive-sensorineural hearing loss manifests as a combination of both sensorineural and conductive loss. Central auditory imperceptive hearing loss includes all hearing losses that do not demonstrate defects in the conduction or sensory structures.

2. Smokeless tobacco is: a. not addicting. b. proven to be carcinogenic. c. easy to stop using. d. a safe alternative to cigarette smoking.

ANS: B Smokeless tobacco is a popular substitute for cigarettes and poses serious health hazards to children and adolescents. Smokeless tobacco is associated with cancer of the mouth and jaw. The nicotine in the smokeless tobacco is addicting, and therefore it is very difficult to quit. Because the product is addicting and can cause cancer, it is not a safe alternative to cigarette smoking.

18. The parents of a neonate with adrenogenital hyperplasia tell the nurse that they are afraid to have any more children. The nurse should explain which statement about adrenogenital hyperplasia? a. It is not hereditary. b. Genetic counseling is indicated. c. It can be prevented during pregnancy. d. All future children will have the disorder.

ANS: B Some forms of adrenogenital hyperplasia are hereditary and should be referred for genetic counseling. Affected offspring should also be referred for genetic counseling. There is an autosomal recessive form of adrenogenital hyperplasia. A prenatal treatment with glucocorticoids can be offered to the mother during pregnancy to avoid the sex ambiguity, but it does not affect the presence of the disease. If it is the heritable form, for each pregnancy, a 25% risk occurs that the child will be affected.

12. The nurse is planning care for an 8-year-old child with a concussion. Which is descriptive of a concussion? a. Petechial hemorrhages cause amnesia. b. Visible bruising and tearing of cerebral tissue occur. c. It is a transient and reversible neuronal dysfunction. d. A slight lesion develops remotely from the site of trauma.

ANS: C A concussion is a transient, reversible neuronal dysfunction with instantaneous loss of awareness and responsiveness resulting from trauma to the head. Petechial hemorrhages along the superficial aspects of the brain along the point of impact are a type of contusion, but are not necessarily associated with amnesia. A contusion is visible bruising and tearing of cerebral tissue. Contrecoup is a lesion that develops remote from the site of trauma as a result of an acceleration-deceleration injury.

2. A child with hypopituitarism is being started on growth hormone (GH) therapy. Nursing considerations should be based on which knowledge? a. Treatment is most successful if it is started during adolescence. b. Treatment is considered successful if children attain full stature by adulthood. c. Replacement therapy requires daily subcutaneous injections. d. Replacement therapy will be required throughout the child's lifetime.

ANS: C Additional support is required for children who require hormone replacement therapy, such as preparation for daily subcutaneous injections and education for self-management during the school-age years. Young children, obese children, and those who are severely GH deficient have the best response to therapy. When therapy is successful, children can attain their actual or near-final adult height at a slower rate than their peers. Replacement therapy is not needed after attaining final height. They are no longer GH deficient

30. To help the adolescent deal with diabetes, the nurse must consider which characteristic of adolescence? a. Desire to be unique b. Preoccupation with the future c. Need to be perfect and similar to peers d. Need to make peers aware of the seriousness of hypoglycemic reactions

ANS: C Adolescence is a time when the individual wants to be perfect and similar to peers. Having diabetes makes adolescents different from their peers. Adolescents do not wish to be unique; they desire to fit in with the peer group and are usually not future oriented. Forcing peer awareness of the seriousness of hypoglycemic reactions would further alienate the adolescent with diabetes. The peer group would focus on the differences.

7. The nurse is caring for a preschool child with suspected diabetes insipidus. Which clinical manifestation should the nurse expect to observe? a. Oliguria b. Glycosuria c. Nausea and vomiting d. Polyuria and polydipsia

ANS: D Excessive urination accompanied by insatiable thirst is the primary clinical manifestation of diabetes. These symptoms may be so severe that the child does little other than drink and urinate. Oliguria is decreased urine production and is not associated with diabetes insipidus. Glycosuria is associated with diabetes mellitus. Nausea and vomiting are associated with inappropriate antidiuretic hormone (ADH) secretion.

3. Which describes the cognitive abilities of school-age children? a. Have developed the ability to reason abstractly b. Are capable of scientific reasoning and formal logic c. Progress from making judgments based on what they reason to making judgments based on what they see d. Are able to classify, to group and sort, and to hold a concept in their minds while making decisions based on that concept

ANS: D In Piaget's stage of concrete operations, children have the ability to group and sort and make conceptual decisions. Children cannot reason abstractly and logically until late adolescence. Making judgments based on what they reason to making judgments based on what they see is not a developmental skill.

27. A school-age child is diagnosed with a life-threatening illness. The parents want to protect their child from knowing the seriousness of the illness. The nurse should explain that: a. this will help the child cope effectively by denial. b. this attitude is helpful to give parents time to cope. c. terminally ill children know when they are seriously ill. d. terminally ill children usually choose not to discuss the seriousness of their illness.

ANS: C The child needs honest and accurate information about the illness, treatments, and prognosis. Children, even at a young age, realize that something is seriously wrong and that it involves them. The nurse should help the parents understand the importance of honesty.

2. Generally, the earliest age at which puberty begins is _____ years in girls, _____ in boys. a. 13; 13 b. 11; 11 c. 10; 12 d. 12; 10

ANS: C Puberty signals the beginning of the development of secondary sex characteristics. This begins earlier in girls than in boys. Usually a 2-year difference occurs in the age of onset. Girls and boys do not usually begin puberty at the same age. Girls generally begin puberty 2 years earlier than boys.

18. The nurse is talking with a 10-year-old boy who wears bilateral hearing aids. The left hearing aid is making an annoying whistling sound that the child cannot hear. Which is the most appropriate nursing action? a. Ignore the sound. b. Ask him to reverse the hearing aids in his ears. c. Suggest he reinsert the hearing aid. d. Suggest he raise the volume of the hearing aid.

ANS: C The whistling sound is acoustic feedback. The nurse should have the child remove the hearing aid and reinsert it, making certain no hair is caught between the ear mold and the ear canal. It would be annoying to others to ignore the sound or to suggest he raise the volume of the hearing aid. The hearing aids are molded specifically for each ear.

5. The nurse is caring for a school-age child with hyperthyroidism (Graves disease). Which clinical manifestations should the nurse monitor that may indicate a thyroid storm? (Select all that apply.) a. Constipation b. Hypotension c. Hyperthermia d. Tachycardia e. Vomiting

ANS: C, D, E A child with a thyroid storm will have severe irritability and restlessness, vomiting, diarrhea, hyperthermia, hypertension, severe tachycardia, and prostration.

5. The nurse is caring for a neonate with suspected meningitis. Which clinical manifestations should the nurse prepare to assess if meningitis is confirmed? (Select all that apply.) a. Headache b. Photophobia c. Bulging anterior fontanel d. Weak cry e. Poor muscle tone

ANS: C, D, E Assessment findings in a neonate with meningitis include bulging anterior fontanel, weak cry, and poor muscle tone. Headache and photophobia are signs seen in an older child.

40. Children taking phenobarbital (phenobarbital sodium) and/or phenytoin (Dilantin) may experience a deficiency of: a. calcium. b. vitamin C. c. fat-soluble vitamins. d. vitamin D and folic acid.

ANS: D Deficiencies of vitamin D and folic acid have been reported in children taking phenobarbital and phenytoin. Calcium, vitamin C, and fat-soluble vitamin deficiencies are not associated with phenobarbital or phenytoin.

20. A school nurse is teaching dental health practices to a group of sixth-grade children. How often should the nurse recommend the children brush their teeth? a. Twice a day b. Three times a day c. After meals d. After meals, snacks, and bedtime

ANS: D Teeth should be brushed after meals, after snacks, and at bedtime. Children who brush their teeth frequently and become accustomed to the feel of a clean mouth at an early age usually maintain the habit throughout life. Twice a day, three times a day or after meals would not be often enough.

19. A toddler fell out of a second-story window. She had a brief loss of consciousness and vomited four times. Since admission, she has been alert and oriented. Her mother asks why a computed tomography (CT) scan is required when she "seems fine." Which explanation should the nurse give? a. Your child may have a brain injury and the CT can rule one out. b. The CT needs to be done because of your child's age. c. Your child may start to have seizures and a baseline CT should be done. d. Your child probably has a skull fracture and the CT can confirm this diagnosis.

ANS: A The child's history of the fall, brief loss of consciousness, and vomiting four times necessitates evaluation of a potential brain injury. The severity of a head injury may not be apparent on clinical examination but will be detectable on a CT scan. The need for the CT scan is related to the injury and symptoms, not the child's age. The CT scan is necessary to determine whether a brain injury has occurred.

14. A sexually active female adolescent asks the nurse about the contraceptive Depo-Provera. The nurse should explain that it: a. requires injections every 3 months. b. requires daily administration of medication by mouth. c. provides long-term continuous protection, up to 5 years. d. prevents pregnancy if given within 72 hours of unprotected sex.

ANS: A The contraceptive Depo-Provera is administered by injection every 3 months. Oral contraceptives, not Depo-Provera, require daily administration of medication by mouth. Norplant, not Depo-Provera, provides long-term continuous protection for up to 5 years. Postcoital contraception, not Depo-Provera, prevents pregnancy if given within 72 hours of unprotected sex.

6. The nurse is taking care of a child who is alert but showing signs of increased intracranial pressure. Which test is contraindicated in this case? a. Oculovestibular response b. Doll's head maneuver c. Funduscopic examination for papilledema d. Assessment of pyramidal tract lesions

ANS: A The oculovestibular response (caloric test) involves the instillation of ice water into the ear of a comatose child. The caloric test is painful and is never performed on an awake child or one who has a ruptured tympanic membrane. Doll's head maneuver, funduscopic examination for papilledema, and assessment of pyramidal tract lesions can be performed on awake children.

16. Which is the most appropriate nursing intervention to promote normalization in a school-age child with a chronic illness? a. Give child as much control as possible. b. Ask child's peer to make child feel normal. c. Convince child that nothing is wrong with him or her. d. Explain to parents that family rules for the child do not need to be the same as for healthy siblings.

ANS: A The school-age child who is ill may be forced into a period of dependency. To foster normalcy, the child should be given as much control as possible. It is unrealistic to expect one individual to make the child feel normal. The child has a chronic illness. It would be unacceptable to convince the child that nothing is wrong. The family rules should be similar for each of the children in a family. Resentment and hostility can arise if different standards are applied to each child.

30. The nurse is talking with the parents of a child who died 6 months ago. They sometimes still "hear" the child's voice and have trouble sleeping. They describe feeling "empty" and depressed. The nurse should recognize that: a. these are normal grief responses. b. the pain of the loss is usually less by this time. c. these grief responses are more typical of the early stages of grief. d. this grieving is essential until the pain is gone and the child is gradually forgotten.

ANS: A These are normal grief responses. The process of grief work is lengthy.

6. Turner syndrome is suspected in an adolescent girl with short stature. This is caused by: a. absence of one of the X chromosomes. b. presence of an incomplete Y chromosome. c. precocious puberty in an otherwise healthy child. d. excess production of both androgens and estrogens.

ANS: A Turner syndrome is caused by an absence of one of the X chromosomes. Most girls who have this disorder have one X chromosome missing from all cells. No Y chromosome is present in individuals with Turner syndrome. This young woman has 45 rather than 46 chromosomes.

28. A cure is no longer possible for a young child with cancer. The nursing staff recognizes that the goal of treatment must shift from cure to palliation. Which is an important consideration at this time? a. The family is included in the decision to shift the goals of treatment. b. The decision must be made by the health professionals involved in the child's care. c. The family needs to understand that palliative care takes place in the home. d. The decision should not be communicated to the family because it will encourage a sense of hopelessness.

ANS: A When the child reaches the terminal stage, the nurse and physician should explore the family's wishes. The family should help decide what interventions will occur as they plan for their child's death.

34. The nurse is caring for an adolescent brought to the hospital with acute drug toxicity. Cocaine is believed to be the drug involved. Data collection should include what information? a. Mode of administration b. Drug's actual content c. Function the drug plays in the adolescent's life d. Adolescent's level of interest in rehabilitation

ANS: A When the drug is questionable or unknown, every effort must be made to determine the type, amount of drug taken, the mode and time of administration, and factors relating to the onset of presenting symptoms. The actual content of most street drugs is highly questionable. Pharmacologic agents should be administered with caution, except for the narcotic antagonists in case of suspected opioid use. The function the drug plays in the adolescent's life and the adolescent's level of interest in rehabilitation are important considerations in the long-term management during the nonacute stage.

4. The nurse is evaluating the laboratory results on cerebral spinal fluid (CSF) from a 3-year-old child with bacterial meningitis. Which findings confirm bacterial meningitis? (Select all that apply.) a. Elevated white blood cell (WBC) count b. Decreased glucose c. Normal protein d. Elevated red blood cell (RBC) count

ANS: A, B The cerebrospinal fluid analysis in bacterial meningitis shows elevated WBC count, decreased glucose, and increased protein content. There should not be RBCs evident in the CSF fluid.

2. Which assessment findings indicate to the nurse a child has Down syndrome? (Select all that apply.) a. High arched narrow palate b. Protruding tongue c. Long, slender fingers d. Transverse palmar crease e. Hypertonic muscle tone

ANS: A, B, D The assessment findings of Down syndrome include high arched narrow palate, protruding tongue, and transverse palmar creases. The fingers are stubby and the muscle tone is hypotonic not hypertonic.

1. A nurse is recommending strategies to a group of school-age children for prevention of obesity. Which should the nurse include? (Select all that apply.) a. Eat breakfast daily. b. Limit fruits and vegetables. c. Have frequent family meals with parents present. d. Eat frequently at restaurants. e. Limit television viewing to 2 hours a day.

ANS: A, C, E The nurse should counsel school-age children to eat breakfast daily, have mealtimes with family, and limit television viewing to 2 hours a day to prevent obesity. Fruits and vegetables should be consumed in the recommended quantities, and eating at restaurants should be limited.

1. The treatment of brain tumors in children consists of which therapies? (Select all that apply.) a. Surgery b. Bone marrow transplantation c. Chemotherapy d. Stem cell transplantation e. Radiation f. Myelography

ANS: A, C, E Treatment for brain tumors in children may consist of surgery, chemotherapy, and radiotherapy alone or in combination. Bone marrow and stem cell transplantation therapies are used for leukemia, lymphoma, and other solid tumors where myeloablative therapies are used. Myelography is a radiographic examination after an intrathecal injection of contrast medium. It is not a treatment.

2. Which are appropriate statements the nurse should make to parents after the death of their child? (Select all that apply.) a. "We feel so sorry that we couldn't save your child." b. "Your child isn't suffering anymore." c. "I know how you feel." d. "You're feeling all the pain of losing a child." e. "You are still young enough to have another baby."

ANS: A, D By saying, "We feel so sorry that we couldn't save your child," the nurse is expressing personal feeling of loss or frustration, which is therapeutic. Stating, "You're feeling all the pain of losing a child," focuses on a feeling, which is therapeutic. The statement, "Your child isn't suffering anymore," is a judgmental statement, which is nontherapeutic. "I know how you feel" and "You're still young enough to have another baby" are statements that give artificial consolation and are nontherapeutic.

4. A nurse is planning interventions for a toddler with juvenile hypothyroidism. Which interventions should the nurse plan to implement for this child? (Select all that apply.) a. Moisturizer for dry skin b. Antidiarrheal medications c. Medications to help with insomnia d. Implementation of thyroxine therapy

ANS: A, D The presenting symptoms of juvenile hypothyroidism are myxedematous skin changes (dry skin, puffiness around the eyes, sparse hair), constipation, lethargy, and mental decline. The nurse should plan interventions for the dry skin and for the implementation of thyroxine therapy. The child is prone to constipation and sleepiness so antidiarrheal medication and medications to help with insomnia would not be appropriate

1. A 6-year-old child is having a generalized seizure in the classroom at school. Place in order the interventions the school nurse should implement starting with the highest-priority intervention sequencing to the lowest-priority intervention. Provide answer using lowercase letters separated by commas (e.g., a, b, c, d, e). a. Take vital signs. b. Ease child to the floor. c. Allow child to rest. d. Turn child to the side. e. Integrate child back into the school environment.

ANS: b, d, a, c, e The nurse should ease the child to the floor immediately during a generalized seizure. During (and sometimes after) the generalized seizure, the swallowing reflex is lost, salivation increases, and the tongue is hypotonic. Therefore, the child is at risk for aspiration and airway occlusion. Placing the child on the side facilitates drainage and helps maintain a patent airway. Vital signs should be taken next and the child should be allowed to rest. When feasible, the child is integrated into the environment as soon as possible.

31. When caring for the child with Reye syndrome, the priority nursing intervention should be to: a. monitor intake and output. b. prevent skin breakdown. c. observe for petechiae. d. do range-of-motion exercises.

ANS: A Accurate and frequent monitoring of intake and output is essential for adjusting fluid volumes to prevent both dehydration and cerebral edema. Preventing skin breakdown, observing for petechiae, and doing range-of-motion exercises are important interventions in the care of a critically ill or comatose child. Careful monitoring of intake and output is a priority.

15. A child has an evulsed (knocked-out) tooth. Which medium should the nurse instruct the parents to place the tooth in for transport to the dentist? a. In cold milk b. In cold water c. In warm salt water d. In a dry, clean jar

ANS: A An evulsed tooth should be placed in a suitable medium for transplant, either cold milk or saliva (under the child or parent's tongue). Cold milk is a more suitable medium for transport than cold water, warm salt water, or a dry, clean jar.

41. Which clinical manifestations would suggest hydrocephalus in a neonate? a. Bulging fontanel and dilated scalp veins b. Closed fontanel and high-pitched cry c. Constant low-pitched cry and restlessness d. Depressed fontanel and decreased blood pressure

ANS: A Bulging fontanels, dilated scalp veins, and separated sutures are clinical manifestations of hydrocephalus in neonates. Closed fontanel and high-pitched cry, constant low-pitched cry and restlessness, and depressed fontanel and decreased blood pressure are not clinical manifestations of hydrocephalus, but all should be referred for evaluation.

31. The nurse is talking to the parent of a 13-month-old child. The mother states, "My child does not make noises like 'da' or 'na' like my sister's baby, who is only 9 months old." Which statement by the nurse would be most appropriate to make? a. "I am going to request a referral to a hearing specialist." b. "You should not compare your child to your sister's child." c. "I think your child is fine, but we will check again in 3 months." d. "You should ask other parents what noises their children made at this age."

ANS: A By 11 months of age a child should be making well-formed syllables such as "da" or "na" and should be referred to a specialist if not. You should not compare your child to your sister's child, I think your child is fine, but we will check again in 3 months, and You should ask other parents what noises their children made at this age are not appropriate statements to make to the parent.

22. Prevention of hearing impairment in children is a major goal for the nurse. This can be achieved through which intervention? a. Being involved in immunization clinics for children b. Assessing a newborn for hearing loss c. Answering parents' questions about hearing aids d. Participating in hearing screening in the community

ANS: A Childhood immunizations can eliminate the possibility of acquired sensorineural hearing loss from rubella, mumps, or measles encephalitis. Assessing a newborn for hearing loss, answering parents' questions about hearing aids, and participating in hearing screening in the community are interventions to screen for the presence of hearing loss or deal with an identified loss, not prevention.

6. An 8-year-old girl tells the nurse that she has cancer because God is punishing her for "being bad." She shares her concern that if she dies, she will go to hell. The nurse should interpret this as: a. a belief common at this age. b. a belief that forms the basis for most religions. c. suggestive of excessive family pressure. d. suggestive of a failure to develop a conscience.

ANS: A Children at this age may view illness or injury as a punishment for a real or imagined misdeed. The belief in divine punishment is common for an 8-year-old child.

1. Which describes avoidance behaviors parents may exhibit when learning that their child has a chronic condition? (Select all that apply.) a. Refuses to agree to treatment b. Shares burden of disorder with others c. Verbalizes possible loss of child d. Withdraws from outside world e. Punishes self because of guilt and shame

ANS: A, D, E A parent who refuses to agree to treatment, withdraws from the outside world, and punishes self because of guilt and shame is exhibiting avoidance coping behaviors. A parent who shares the burden of disorder with others and verbalizes possible loss of child is exhibiting approach coping behaviors.

2. The nurse should expect to assess which clinical manifestations in an adolescent with Cushing syndrome? (Select all that apply.) a. Hyperglycemia b. Hyperkalemia c. Hypotension d. Cushingoid features e. Susceptibility to infections

ANS: A, D, E In Cushing syndrome, physiologic disturbances seen are Cushingoid features hyperglycemia, susceptibility to infection, hypertension, and hypokalemia.

1. Which statement is true about smoking in adolescence? a. Smoking is related to other high-risk behaviors. b. Smoking will not continue unless peer pressure continues. c. Smoking is less common when the adolescent's parent(s) smokes. d. Smoking among adolescents is becoming more prevalent.

ANS: A Cigarettes are considered a gateway drug. Teenagers who smoke are 11.4 times more likely to use an illicit drug. Teenagers begin smoking for a variety of reasons, such as imitation of adult behavior, peer pressure, imitation of behaviors portrayed in movies and advertisements, and a desire to control weight. The absence of peer pressure alone will not stop smoking. Teenagers who do not smoke usually have parents and friends who do not smoke or who oppose smoking. The percentage of young people who report current cigarette use and frequent cigarette use has declined significantly.

3. When should children with cognitive impairment be referred for stimulation and educational programs? a. As young as possible b. As soon as they have the ability to communicate in some way c. At age 3 years, when schools are required to provide services d. At age 5 or 6 years, when schools are required to provide services

ANS: A The child's education should begin as soon as possible. Considerable evidence exists that early intervention programs for children with disabilities are valuable for cognitively impaired children. The early intervention may facilitate the child's development of communication skills. States are encouraged to provide early intervention programs from birth under Public Law 101-476, the Individuals with Disabilities Education Act.

1. Autism is a complex developmental disorder. The diagnostic criteria for autism include delayed or abnormal functioning in which areas with onset before age 3 years? (Select all that apply.) a. Language as used in social communication b. Parallel play c. Gross motor development d. Growth below the 5th percentile for height and weight e. Symbolic or imaginative play f. Social interaction

ANS: A, E, F These are three of the areas in which autistic children may show delayed or abnormal functioning: language as used in social communication, symbolic or imaginative play, and social interaction. Parallel play is typical play of toddlers and is usually not affected. Gross motor development and growth below the 5th percentile for height and weight are usually not characteristic of autism.

13. The nurse is teaching nursing students about childhood fractures. Which describes a compound skull fracture? a. Involves the basilar portion of the occipital bone b. Bone is exposed through the skin c. Traumatic separations of the cranial sutures d. Bone is pushed inward, causing pressure on the brain

ANS: B A compound fracture has the bone exposed through the skin. A basilar fracture involves the basilar portion of the frontal, ethmoid, sphenoid, temporal, or occipital bone. Diastatic skull fractures are traumatic separations of the cranial sutures. A depressed fracture has the bone pushed inward, causing pressure on the brain.

32. A young child's parents call the nurse after their child was bitten by a raccoon in the woods. The nurse's recommendation should be based on which statement? a. Child should be hospitalized for close observation. b. No treatment is necessary if thorough wound cleaning is done. c. Antirabies prophylaxis must be initiated. d. Antirabies prophylaxis must be initiated if clinical manifestations appear.

ANS: C Current therapy for a rabid animal bite consists of a thorough cleansing of the wound and passive immunization with human rabies immune globulin (HRIG) as soon as possible. Hospitalization is not necessary. The wound cleansing, passive immunization, and immune globulin administration can be done as an outpatient. The child needs to receive both HRIG and rabies vaccine.

25. The parents of a child who has just been diagnosed with type 1 diabetes ask about exercise. Which should the nurse explain about exercise in type 1 diabetes? a. Exercise will increase blood glucose. b. Exercise should be restricted. c. Extra snacks are needed before exercise. d. Extra insulin is required during exercise.

ANS: C Exercise lowers blood glucose levels, which can be compensated for by extra snacks. Exercise lowers blood glucose and is encouraged and not restricted, unless indicated by other health conditions. Extra insulin is contraindicated because exercise decreases blood glucose levels

13. A child has just been diagnosed with fragile X syndrome. The nurse recognizes that fragile X syndrome is: a. a chromosomal defect affecting females only. b. a chromosomal defect that follows the pattern of X-linked recessive disorders. c. the second most common genetic cause of mental retardation. d. the most common cause of noninherited mental retardation.

ANS: C Fragile X syndrome is the second most common cause of mental retardation after Down syndrome. Fragile X primarily affects males, follows the inheritance pattern of X-linked dominant with reduced penetrance. This is in distinct contrast to the classic X-linked recessive pattern in which all carrier females are normal, all affected males have symptoms of the disorder, and no males are carriers.

1. Parents of a toddler with hypopituitarism ask the nurse, "What can we expect with this condition?" The nurse should respond with which statement? a. Growth is normal during the first 3 years of life. b. Weight is usually more retarded than height. c. Skeletal proportions are normal for age. d. Most of these children have subnormal intelligence.

ANS: C In children with hypopituitarism, the skeletal proportions are normal. Growth is within normal limits for the first year of life. Height is usually more delayed than weight. Intelligence is not affected by hypopituitarism.

21. Young people with anorexia nervosa are often described as being: a. independent. b. disruptive. c. conforming. d. low achieving.

ANS: C Individuals with anorexia nervosa are described as perfectionist, academically high achievers, conforming, and conscientious. "Independent," disruptive," and "low achieving" are not part of the behavioral characteristics of anorexia nervosa.

4. A 12-year-old male has short stature because of a constitutional growth delay. The nurse should be the most concerned about which of the following? a. Proper administration of thyroid hormone b. Proper administration of human growth hormones c. Child's self-esteem and sense of competence d. Helping child understand that his height is most likely caused by chronic illness and is not his fault

ANS: C Most cases of constitutional growth delay are caused by simple constitutional delay of puberty, and the child can be assured that normal development will eventually take place. Listening to distressed adolescents and conveying interest and concern are important interventions for these children and adolescents. They should be encouraged to focus on the positives aspects of their bodies and personalities. Thyroid hormones and human growth hormones would not be beneficial in a constitutional growth delay. A constitutional growth delay is not caused by a chronic illness.

11. An appropriate nursing intervention when caring for an unconscious child should be to: a. change the child's position infrequently to minimize the chance of increased ICP. b. avoid using narcotics or sedatives to provide comfort and pain relief. c. monitor fluid intake and output carefully to avoid fluid overload and cerebral edema. d. give tepid sponge baths to reduce fever because antipyretics are contraindicated.

ANS: C Often comatose patients cannot cope with the quantity of fluids that they normally tolerate. Overhydration must be avoided to prevent fatal cerebral edema. The child's position should be changed frequently to avoid complications such as pneumonia and skin breakdown. Narcotics and sedatives should be used as necessary to reduce pain and discomfort, which can increase ICP. Antipyretics are the method of choice for fever reduction.

22. A nurse planning care for a school-age child should take into account that which thought process is seen at this age? a. Animism b. Magical thinking c. Ability to conserve d. Thoughts are all-powerful

ANS: C One cognitive task of school-age children is mastering the concept of conservation. At an early age (5 to 7 years), children grasp the concept of reversibility of numbers as a basis for simple mathematics problems (e.g., 2 + 4 = 6 and 6 - 4 = 2). They learn that simply altering their arrangement in space does not change certain properties of the environment, and they are able to resist perceptual cues that suggest alterations in the physical state of an object. Animism, magical thinking, and believing that thoughts are all powerful are thought processes seen in preschool children.

8. A group of boys ages 9 and 10 years have formed a "boys-only" club that is open to neighborhood and school friends who have skateboards. This should be interpreted as: a. behavior that encourages bullying and sexism. b. behavior that reinforces poor peer relationships. c. characteristic of social development at this age. d. characteristic of children who later are at risk for membership in gangs.

ANS: C One of the outstanding characteristics of middle childhood is the creation of formalized groups or clubs. Peer-group identification and association are essential to a child's socialization. Poor relationships with peers and a lack of group identification can contribute to bullying. A boys-only club does not have a direct correlation with later gang activity

24. Which of the following terms refers to opacity of the crystalline lens that prevents light rays from entering the eye and refracting on the retina? a. Myopia b. Amblyopia c. Cataract d. Glaucoma

ANS: C Opacity of the crystalline lens that prevents light rays from entering the eye and refracting on the retina is the definition of a cataract. Myopia, or nearsightedness, refers to the ability to see objects clearly at close range but not at a distance. Amblyopia, or lazy eye, is reduced visual acuity in one eye. Glaucoma is a group of eye diseases characterized by increased intraocular pressure.

28. Which is an important consideration when the nurse is discussing enuresis with the parents of a young child? a. Enuresis is more common in girls than in boys. b. Enuresis is neither inherited nor has a familial tendency. c. Organic causes that may be related to enuresis should be considered first. d. Psychogenic factors that cause enuresis persist into adulthood.

ANS: C Organic causes that may be related to enuresis should be ruled out before psychogenic factors are considered. Enuresis is more common in boys than in girls and has a strong familial tendency. Psychogenic factors may influence enuresis, but it is doubtful that they are causative.

17. A nurse is conducting a class for adolescent girls about pelvic inflammatory disease (PID). Why should the nurse emphasize the importance of preventing pelvic inflammatory disease (PID)? a. PID can be sexually transmitted. b. PID cannot be treated. c. PID can have devastating effects on the reproductive tract. d. PID can cause serious defects in future children of affected adolescents.

ANS: C PID is a major concern because of its devastating effects on the reproductive tract. Short-term complications include abscess formation in the fallopian tubes, whereas long-term complications include ectopic pregnancy, infertility, and dyspareunia. PID is an infection of the upper female genital tract, most commonly caused by sexually transmitted infections but it is not sexually transmitted to another person. PID can be treated by treating the underlying cause. There is a possibility of ectopic pregnancy but not birth defects in children.

31. Parents have a concern that their child is depressed. The nurse relates that which characteristic best describes children with depression? a. Increased range of affective response b. Preoccupation with need to perform well in school c. Change in appetite, resulting in weight loss or gain d. Tendency to prefer play instead of schoolwork

ANS: C Physiologic characteristics of children with depression include change in appetite resulting in weight loss or gain, nonspecific complaints of not feeling well, alterations in sleeping pattern, insomnia or hypersomnia, and constipation. Children who are depressed have sad facial expressions with absence or diminished range of affective response. These children withdraw from previously enjoyed activities and engage in solitary play or work with a lack of interest in play. A lack of interest is seen in doing homework or achieving in school, resulting in lower grades in children who are depressed.

20. A 16-year-old boy with a chronic illness has recently become rebellious and is taking risks such as missing doses of his medication. The nurse should explain to his parents that: a. he needs more discipline. b. he needs more socialization with peers. c. this is part of normal adolescence. d. this is how he is asking for more parental control.

ANS: C Risk taking, rebelliousness, and lack of cooperation are normal parts of adolescence.

33. A child is brought to the emergency department after experiencing a seizure at school. There is no previous history of seizures. The father tells the nurse that he cannot believe the child has epilepsy. The nurse's best response is: a. "Epilepsy is easily treated." b. "Very few children have actual epilepsy." c. "The seizure may or may not mean that your child has epilepsy." d. "Your child has had only one convulsion; it probably won't happen again."

ANS: C Seizures are the indispensable characteristic of epilepsy; however, not every seizure is epileptic. Epilepsy is a chronic seizure disorder with recurrent and unprovoked seizures. The treatment of epilepsy involves a thorough assessment to determine the type of seizure the child is having and the cause, followed by individualized therapy to allow the child to have as normal a life as possible. The nurse should not make generalized comments regarding the incidence of epilepsy until further assessment is made.

13. A 9-year-old boy has several physical disabilities. His father explains to the nurse that his son concentrates on what he can, rather than cannot, do and is as independent as possible. The nurse's best interpretation of this is that the: a. father is experiencing denial. b. father is expressing his own views. c. child is using an adaptive coping style. d. child is using a maladaptive coping style.

ANS: C The father is describing a well-adapted child who has learned to accept physical limitations. These children function well at home, at school, and with peers. They have an understanding of their disorder that allows them to accept their limitations, assume responsibility for care, and assist in treatment and rehabilitation. The father is describing his child's behavior. He is not denying the child's limitations. The father is exhibiting an adaptive coping style.

28. A father calls the emergency department nurse saying that his daughter's eyes burn after getting some dishwasher detergent in them. The nurse recommends that the child be seen in the emergency department or by an ophthalmologist. The nurse also should recommend which action before the child is transported? a. Keep eyes closed. b. Apply cold compresses. c. Irrigate eyes copiously with tap water for 20 minutes. d. Prepare a normal saline solution (salt and water) and irrigate eyes for 20 minutes.

ANS: C The first action is to flush the eyes with clean tap water. This will rinse the detergent from the eyes. Keeping eyes closed and applying cold compresses may allow the detergent to do further harm to the eyes during transport. Normal saline is not necessary. The delay can allow the detergent to cause continued injury to the eyes.

7. The nurse observes that a seriously ill child passively accepts all painful procedures. The nurse should recognize that this is most likely an indication that the child is experiencing a: a. sense of hopefulness. b. sense of chronic sorrow. c. belief that procedures are a deserved punishment. d. belief that procedures are an important part of care.

ANS: C The nurse should be particularly alert to the child who passively accepts all painful procedures. This child may believe that such acts are inflicted as deserved punishment. The child who is hopeful is mobilized into goal-directed actions. This child would actively participate in care. Chronic sorrow is the feeling of sorrow and loss that recurs in waves over time. It is usually evident in the parents, not in the child. A child who believes that procedures are an important part of care would actively participate in care. Nursing interventions should be used to minimize the pain.

22. A 5-year-old girl sustained a concussion when she fell out of a tree. In preparation for discharge, the nurse is discussing home care with her mother. Which statement made by the mother indicates a correct understanding of the teaching? a. "I should expect my child to have a few episodes of vomiting." b. "If I notice sleep disturbances, I should contact the physician immediately." c. "I should expect my child to have some behavioral changes after the accident." d. "If I notice diplopia, I will have my child rest for 1 hour."

ANS: C The parents are advised of probable posttraumatic symptoms that may be expected. These include behavioral changes and sleep disturbances. If the child has these clinical signs, they should be immediately reported for evaluation. Sleep disturbances are to be expected.

15. Glucocorticoids, mineralocorticoids, and sex steroids are secreted by the: a. thyroid gland. b. parathyroid glands. c. adrenal cortex. d. anterior pituitary.

ANS: C These hormones are secreted by the adrenal cortex. The thyroid gland produces thyroid hormone and thyrocalcitonin. The parathyroid gland produces parathyroid hormone. The anterior pituitary produces hormones such as GH, thyroid-stimulating hormone, adrenocorticotropic hormone, gonadotropin, prolactin, and melanocyte-stimulating hormone.

9. The school nurse is discussing testicular self-examination with adolescent boys. Why is this important? a. Epididymitis is common during adolescence. b. Asymptomatic sexually transmitted diseases may be present. c. Testicular tumors during adolescence are generally malignant. d. Testicular tumors, although usually benign, are common during adolescence.

ANS: C Tumors of the testes are not common, but when manifested in adolescence, they are generally malignant and demand immediate evaluation. Epididymitis is not common in adolescence. Asymptomatic sexually transmitted disease would not be evident during testicular self-examination. The focus of this examination is on testicular cancer. Testicular tumors are most commonly malignant.

14. A child with hypoparathyroidism is receiving vitamin D therapy. The parents should be advised to watch for which sign of vitamin D toxicity? a. Headache and seizures b. Physical restlessness and voracious appetite without weight gain c. Weakness and lassitude d. Anorexia and insomnia

ANS: C Vitamin D toxicity can be a serious consequence of therapy. Parents are advised to watch for signs, including weakness, fatigue, lassitude, headache, nausea, vomiting, and diarrhea. Renal impairment is manifested through polyuria, polydipsia, and nocturia. Headaches may be a sign of vitamin D toxicity, but seizures are not. Physical restlessness and a voracious appetite with weight loss are manifestations of hyperthyroidism. Anorexia and insomnia are not characteristic of vitamin D toxicity.

30. Which of the following is the most common clinical manifestation of retinoblastoma? a. Glaucoma b. Amblyopia c. Cat's eye reflex d. Sunken eye socket

ANS: C When the eye is examined, the light will reflect off the tumor, giving the eye a whitish appearance. This is called a cat's eye reflex. A late sign of retinoblastoma is a red, painful eye with glaucoma. Amblyopia, or lazy eye, is reduced visual acuity in one eye. The eye socket is not sunken.

3. Which are adaptive coping patterns used by children with special needs? (Select all that apply.) a. Feels different and withdraws b. Is irritable, moody, and acts out c. Seeks support d. Develops optimism

ANS: C, D Adaptive coping patterns used by children with special needs include seeking support and developing optimism. Maladaptive behaviors are seeing themselves as different and withdrawing and becoming irritable, moody, and beginning to act out.

3. A nurse is planning care for a school-age child with type 1 diabetes. Which insulin preparations are rapid and short acting? (Select all that apply.) a. Novolin N b. Lantus c. NovoLog d. Novolin R

ANS: C, D Rapid-acting insulin (e.g., NovoLog) reaches the blood within 15 minutes after injection. The insulin peaks 30 to 90 minutes later and may last as long as 5 hours. Short-acting (regular) insulin (e.g., Novolin R) usually reaches the blood within 30 minutes after injection. The insulin peaks 2 to 4 hours later and stays in the blood for about 4 to 8 hours. Intermediate-acting insulins (e.g., Novolin N) reach the blood 2 to 6 hours after injection. The insulins peak 4 to 14 hours later and stay in the blood for about 14 to 20 hours. Long-acting insulin (e.g., Lantus) takes 6 to 14 hours to start working. It has no peak or a very small peak 10 to 16 hours after injection. The insulin stays in the blood between 20 and 24 hours.

29. Which is beneficial in reducing the risk of Reye syndrome? a. Immunization against the disease b. Medical attention for all head injuries c. Prompt treatment of bacterial meningitis d. Avoidance of aspirin to treat fever associated with influenza

ANS: D Although the etiology of Reye syndrome is obscure, most cases follow a common viral illness, either varicella or influenza. A potential association exists between aspirin therapy and the development of Reye syndrome, so use of aspirin is avoided. No immunization currently exists for Reye syndrome. Reye syndrome is not correlated with head injuries or bacterial meningitis.

29. The nurse is assisting the family of a child with a history of encopresis. Which should be included in the nurse's discussion with this family? a. Instruct the parents to sit the child on the toilet at twice-daily routine intervals. b. Instruct the parents that the child will probably need to have daily enemas. c. Suggest the use of stimulant cathartics weekly. d. Reassure the family that most problems are resolved successfully, with some relapses during periods of stress.

ANS: D Children may be unaware of a prior sensation and unable to control the urge once it begins. They may be so accustomed to bowel accidents that they are unable to smell or feel it. Family counseling is directed toward reassurance that most problems resolve successfully, although relapses during periods of stress are possible. Sitting the child on the toilet is not recommended because it may intensify the parent-child conflict. Enemas may be needed for impactions, but long-term use prevents the child from assuming responsibility for defecation. Stimulant cathartics may cause cramping that can frighten child.

17. The school nurse is conducting a class on bicycle safety. Which statement made by a participant indicates a need for further teaching? a. "Most bicycle injuries occur from a fall off the bicycle." b. "Head injuries are the major causes of bicycle-related fatalities." c. "I should replace my helmet every 5 years." d. "I can ride double with a friend if the bicycle has an extra large seat."

ANS: D Children should not ride double. Most injuries result from falls. The most important aspect of bicycle safety is to encourage the rider to use a protective helmet. Head injuries are the major cause of bicycle-related fatalities. The child should always wear a properly fitted helmet approved by the U.S. Consumer Product Safety Commission and should replace the helmet at least every 5 years.

21. A nurse is preparing a teaching session for parents on prevention of childhood hearing loss. The nurse should include that the most common cause of hearing impairment in children is: a. auditory nerve damage. b. congenital ear defects. c. congenital rubella. d. chronic otitis media.

ANS: D Chronic otitis media is the most common cause of hearing impairment in children. It is essential that appropriate measures be instituted to treat existing infections and prevent recurrences. Auditory nerve damage, congenital ear defects, and congenital rubella are rarer causes of hearing impairment.

10. Most parents of children with special needs tend to experience chronic sorrow. This is characterized by: a. lack of acceptance of child's limitation. b. lack of available support to prevent sorrow. c. periods of intensified sorrow when experiencing anger and guilt. d. periods of intensified sorrow and loss that occur in waves over time.

ANS: D Chronic sorrow is manifested by feelings of sorrow and loss that recur in waves over time. The sorrow is in response to the recognition of the child's limitations. The family should be assessed in an ongoing manner to provide appropriate support as the needs of the family change. The sorrow is not preventable. The chronic sorrow occurs during the reintegration and acknowledgment stage.

21. Hyperglycemia associated with diabetic ketoacidosis is defined as a blood glucose measurement equal to or greater than _____ mg/dl. a. 100 b. 120 c. 180 d. 200

ANS: D Diabetic ketoacidosis is a state of relative insulin insufficiency and may include the presence of hyperglycemia, a blood glucose level greater than or equal to 200 mg/dl. The values 100 mg/dl, 120 mg/dl, and 180 mg/dl are too low for the definition of ketoacidosis.

8. An adolescent girl asks the school nurse for advice because she has dysmenorrhea. She says that a friend recommended she try an over-the-counter nonsteroidal anti-inflammatory drug (NSAID). The nurse's response should be based on which statement? a. Aspirin is the drug of choice for the treatment of dysmenorrhea. b. Over-the-counter NSAIDs are rarely strong enough to provide adequate pain relief. c. NSAIDs are effective because of their analgesic effect. d. NSAIDs are effective because they inhibit prostaglandins, leading to reduction in uterine activity.

ANS: D First-line therapy for adolescents with dysmenorrhea is NSAIDs. This group of drugs blocks the formation of prostaglandins. NSAIDs, not aspirin, are the drugs of choice in dysmenorrhea. NSAIDs are potent anti-inflammatory agents that inhibit prostaglandin. Although NSAIDs have analgesic effects, the mechanism of action in dysmenorrhea is most likely the antiprostaglandin effect.

26. A child eats some sugar cubes after experiencing symptoms of hypoglycemia. This rapid-releasing sugar should be followed by: a. fat. b. fruit juice. c. several glasses of water. d. complex carbohydrate and protein.

ANS: D Symptoms of hypoglycemia are treated with a rapid-releasing sugar source followed by a complex carbohydrate and protein. Fat, fruit juice, and several glasses of water do not provide the child with complex carbohydrate and protein necessary to stabilize the blood glucose.

3. The nurse is performing a Glasgow Coma Scale on a school-age child with a head injury. The child opens eyes spontaneously, obeys commands, and is oriented to person, time, and place. Which is the score the nurse should record? a. 8 b. 11 c. 13 d. 15

ANS: D The Glasgow Coma Scale (GCS) consists of a three-part assessment: eye opening, verbal response, and motor response. Numeric values of 1 through 5 are assigned to the levels of response in each category. The sum of these numeric values provides an objective measure of the patient's level of consciousness (LOC). A person with an unaltered LOC would score the highest, 15. The child who opens eyes spontaneously, obeys commands, and is oriented is scored at a 15.

29. The nurse is discussing with a child and family the various sites used for insulin injections. Which site usually has the fastest rate of absorption? a. Arm b. Leg c. Buttock d. Abdomen

ANS: D The abdomen has the fastest rate of absorption but the shortest duration. The arm has a fast rate of absorption but short duration. The leg has a slow rate of absorption but a long duration. The buttock has the slowest rate of absorption and the longest duration.

2. When a child with mild cognitive impairment reaches the end of adolescence, which characteristic should be expected? a. Achieves a mental age of 5 to 6 years b. Achieves a mental age of 8 to 12 years c. Unable to progress in functional reading or arithmetic d. Acquires practical skills and useful reading and arithmetic to an eighth-grade level

ANS: B By the end of adolescence, the child with mild cognitive impairment can acquire practical skills and useful reading and arithmetic to a third- to sixth-grade level. A mental age of 8 to 12 years is obtainable, and the child can be guided toward social conformity. The achievement of a mental age of 5 to 6 years and being unable to progress in functional reading or arithmetic are characteristics of children with moderate cognitive impairment. Acquiring practical skills and useful reading and arithmetic to an eighth-grade level is not descriptive of cognitive impairment.

28. The vector reservoir for agents causing viral encephalitis in the United States is: a. tarantula spiders. b. mosquitoes. c. carnivorous wild animals. d. domestic and wild animals.

ANS: B Viral encephalitis, not attributable to a childhood viral disease, is usually transmitted by mosquitoes. The vector reservoir for most agents pathogenic for humans and detected in the United States are mosquitoes and ticks; therefore, most cases of encephalitis appear during the hot summer months. Tarantula spiders, carnivorous wild animals, and domestic and wild animals are not reservoirs for the agents that cause viral encephalitis.

18. When teaching injury prevention during the school-age years, what should the nurse include? a. Teach children to fear strangers. b. Teach basic rules of water safety. c. Avoid letting child cook in microwave ovens. d. Caution child against engaging in competitive sports.

ANS: B Water safety instruction is an important source of injury prevention at this age. The child should be taught to swim, select safe and supervised places to swim, swim with a companion, check for sufficient water depth before diving, and use an approved flotation device. Teach stranger safety, not fear of strangers. This includes instructing children to not go with strangers, not wear personalized clothing in public places, tell parents if anyone makes child feel uncomfortable, and say "no" in uncomfortable situations. Teach child safe cooking. Caution against engaging in hazardous sports such as those involving trampolines.

11. The child with Down syndrome should be evaluated for which condition before participating in some sports? a. Hyperflexibility b. Cutis marmorata c. Atlantoaxial instability d. Speckling of iris (Brushfield spots)

ANS: C Children with Down syndrome are at risk for atlantoaxial instability. Before participating in sports that put stress on the head and neck, a radiologic examination should be done. Hyperflexibility, cutis marmorata, and speckling of iris (Brushfield spots) are characteristic of Down syndrome, but they do not affect the child's ability to participate in sports.

12. Many of the physical characteristics of Down syndrome present nursing problems. Care of the child should include which intervention? a. Delay feeding solid foods until the tongue thrust has stopped. b. Modify diet as necessary to minimize the diarrhea that often occurs. c. Provide calories appropriate to child's age. d. Use a cool-mist vaporizer to keep mucous membranes moist.

ANS: D The constant stuffy nose forces the child to breathe by mouth, drying the mucous membranes and increasing the susceptibility to upper respiratory tract infections. A cool-mist vaporizer will keep the mucous membranes moist and liquefy secretions. The child has a protruding tongue, which makes feeding difficult. The parents must persist with feeding while the child continues the physiologic response of the tongue thrust. The child is predisposed to constipation. Calories should be appropriate to the child's weight and growth needs, not age

20. Anorexia nervosa may best be described as: a. occurring most frequently in adolescent males. b. occurring most frequently in adolescents from lower socioeconomic groups. c. resulting from a posterior pituitary disorder. d. resulting in severe weight loss in the absence of obvious physical causes.

ANS: D The etiology of anorexia remains unclear, but a distinct psychological component is present. The diagnosis is based primarily on psychological and behavioral criteria. Females account for 90% to 95% of the cases. No relation has been identified between socioeconomic groups and anorexia. Posterior pituitary disorders are not associated with anorexia nervosa.

7. An adolescent asks the nurse what causes primary dysmenorrhea. The nurse's response should be based on which statement? a. It is an inherited problem. b. Excessive estrogen production causes uterine pain. c. There is no physiologic cause; it is a psychological reaction. d. There is a relation between prostaglandins and uterine contractility.

ANS: D The exact etiology of primary dysmenorrhea is debated. Overproduction of uterine prostaglandins has been implicated, as has overproduction of vasopressin. Dysmenorrhea is not known to be inherited. Excessive estrogen has not been implicated in the etiology. It has a physiologic cause. Women with dysmenorrhea have higher prostaglandin levels.

20. The nurse is assessing a child who was just admitted to the hospital for observation after a head injury. Which is the most essential part of the nursing assessment to detect early signs of a worsening condition? a. Posturing b. Vital signs c. Focal neurologic signs d. Level of consciousness

ANS: D The most important nursing observation is assessment of the child's level of consciousness. Alterations in consciousness appear earlier in the progression of an injury than do alterations of vital signs or focal neurologic signs. Neurologic posturing is indicative of neurologic damage. Vital signs and focal neurologic signs are later signs of progression when compared with level-of-consciousness changes.

2. Which are clinical manifestations of increased intracranial pressure (ICP) in infants? (Select all that apply.) a. Low-pitched cry b. Sunken fontanel c. Diplopia and blurred vision d. Irritability e. Distended scalp veins f. Increased blood pressure

ANS: D, E Diplopia and blurred vision, irritability, and distended scalp veins are signs of increased ICP in infants. Diplopia and blurred vision is indicative of ICP in children. A high-pitched cry and a tense or bulging fontanel are characteristics of increased ICP. Increased blood pressure, common in adults, is rarely seen in children.

3. Which expected appearance will the nurse explain to parents of an infant returning from surgery after an enucleation was performed to treat retinoblastoma? (Select all that apply.) a. A lot of drainage will come from the affected socket. b. The face may be edematous or ecchymotic. c. The eyelids will be sutured shut for the first week. d. There will be an eye pad dressing taped over the surgical site. e. The implanted sphere is covered with conjunctiva and resembles the lining of the mouth.

NS: B, D, E After enucleation surgery, the parents are prepared for the child's facial appearance. An eye patch is in place, and the child's face may be edematous or ecchymotic. Parents often fear seeing the surgical site because they imagine a cavity in the skull. A surgically implanted sphere maintains the shape of the eyeball, and the implant is covered with conjunctiva. When the eyelids are open, the exposed area resembles the mucosal lining of the mouth. The dressing, consisting of an eye pad taped over the surgical site, is changed daily. The wound itself is clean and has little or no drainage. So expecting a lot of drainage is not accurate to tell parents. The eyelids are not sutured shut after enucleation surgery.

16. Which statement regarding chlamydia infection is correct? a. Treatment of choice is oral penicillin. b. Treatment of choice is nystatin or miconazole. c. Clinical manifestations include dysuria and urethral itching in males. d. Clinical manifestations include small, painful vesicles on genital areas.

NS: C Symptoms of chlamydia infection in males include meatal erythema, tenderness, itching, dysuria, and urethral discharge. Some infected males have no symptoms. Oral penicillin and nystatin or miconazole are not the antibiotics of choice. Small, painful vesicles on genital areas are clinical manifestations true of chlamydia infection but may also indicate herpetic lesions.

10. A goiter is an enlargement or hypertrophy of which gland? a. Thyroid b. Adrenal c. Anterior pituitary d. Posterior pituitary

ANS: A A goiter is an enlargement or hypertrophy of the thyroid gland. Goiter is not associated with the adrenal, anterior pituitary, or posterior pituitary organs.

16. Hearing is expressed in decibels (dB), or units of loudness. Which is the softest sound a normal ear can hear? a. 0 dB b. 10 dB c. 40 to 50 dB d. 100 dB

ANS: A By definition, 0 dB is the softest sound the normal ear can hear. Ten dB is the sound of the heartbeat or the rustling of leaves. 40 to 50 dB is in the range of normal conversation. The noise of a train is approximately 100 dB.

3. A child with growth hormone (GH) deficiency is receiving GH therapy. When is the best time for the GH to be administered? a. At bedtime b. After meals c. Before meals d. On arising in the morning

ANS: A Injections are best given at bedtime to more closely approximate the physiologic release of GH. After or before meals and on arising in the morning do not mimic the physiologic release of the hormone.

11. A 14-year-old boy and his parents are concerned about bilateral breast enlargement. The nurse's discussion of this should be based on which statement? a. This is usually benign and temporary. b. This is usually caused by Klinefelter syndrome. c. Administration of estrogen effectively reduces gynecomastia. d. Administration of testosterone effectively reduces gynecomastia.

ANS: A The male breast responds to hormonal changes. Some degree of bilateral or unilateral breast enlargement occurs frequently in boys during puberty. Although individuals with Klinefelter syndrome can have gynecomastia, it is not a common cause for male breast enlargement. Estrogen is not a therapy for gynecomastia. Administration of testosterone has no benefit for gynecomastia and may aggravate the condition.

29. The nurse is caring for a child who has just died. The parents ask to be left alone so that they can rock their child one more time. The nurse's most appropriate response is to: a. grant their request. b. assess why they feel this is necessary. c. discourage this because it will only prolong their grief. d. kindly explain that they need to say good-bye to their child now and leave.

ANS: A The parents should be allowed to remain with their child after the death. The nurse can remove all the tubes and equipment and offer the parents the option of preparing the body.

25. Which is most descriptive of a school-age child's reaction to death? a. Is very interested in funerals and burials b. Has little understanding of words such as forever c. Imagines the deceased person to be still alive d. Has an idealistic view of world and criticizes funerals as barbaric

ANS: A The school-age child is interested in post-death services and may be inquisitive about what happens to the body.

1. A nurse is planning care for a 7-year-old child hospitalized with osteomyelitis. Which activities should the nurse plan to bring from the playroom for the child? (Select all that apply.) a. Paper and some paints b. Board games c. Jack-in-the-box d. Stuffed animals e. Computer games

ANS: A, B, E School-age children become fascinated with complex board, card, or computer games that they can play alone, with a best friend, or with a group. They also enjoy sewing, cooking, carpentry, gardening, and creative activities such as painting. Jack-in-the-box and stuffed animals would be appropriate for a toddler or preschool child.

16. Chronic adrenocortical insufficiency is also referred to as: a. Graves disease. b. Addison disease. c. Cushing syndrome. d. Hashimoto disease.

ANS: B Addison disease is chronic adrenocortical insufficiency. Graves and Hashimoto diseases involve the thyroid gland. Cushing syndrome is a result of excessive circulation of free cortisol.

19. Which is an implanted ear prosthesis for children with sensorineural hearing loss? a. Hearing aid b. Cochlear implant c. Auditory implant d. Amplification device

ANS: B Cochlear implants are surgically implanted, and they provide a sensation of hearing for individuals who have severe or profound hearing loss of sensorineural origin. Hearing aids and amplification devices are external devices for enhancing hearing. Auditory implants do not exist.

7. The nurse is preparing a school-age child for computed tomography (CT scan) to assess cerebral function. The nurse should include which statement in preparing the child? a. "Pain medication will be given." b. "The scan will not hurt." c. "You will be able to move once the equipment is in place." d. "Unfortunately, no one can remain in the room with you during the test."

ANS: B For CT scans, the child must be immobilized. It is important to emphasize to the child that at no time is the procedure painful. Pain medication is not required; however, sedation is sometimes necessary. Someone is able to remain with the child during the procedure.

30. When taking the history of a child hospitalized with Reye syndrome, the nurse should not be surprised that a week ago the child had recovered from: a. measles. b. varicella. c. meningitis. d. hepatitis.

ANS: B Most cases of Reye syndrome follow a common viral illness such as varicella or influenza. Measles, meningitis, and hepatitis are not associated with Reye syndrome.

39. An adolescent has been diagnosed with lactose maldigestion intolerance. The nurse teaches the adolescent about lactose maldigestion intolerance and notes the teen needs further teaching if which statement is made? a. "I will limit my milk consumption to one to two glasses a day." b. "I should drink the milk alone and not with other foods." c. "Hard cheese, cottage cheese, or yogurt can be substituted for milk." d. "I will take a calcium supplement daily."

ANS: B Most people are able to tolerate small amounts of lactose ( 1 cup of milk per day) even in the presence of deficient lactase activity. It is recommended that individuals with lactose maldigestion who do not experience lactose intolerance symptoms continue to consume small amounts of dairy products with meals to prevent reduced bone mass density and subsequent osteoporosis. Hard cheese, cottage cheese, and yogurt are sources of lactose that may be better tolerated. A calcium supplement should be taken daily. Milk taken at meals may be better tolerated than when taken alone.

34. The nurse and a new nurse are caring for a child who will require palliative care. Which statement made by the new nurse would indicate a correct understanding of palliative care? a. "Palliative care serves to hasten death and make the process easier for the family." b. "Palliative care provides pain and symptom management for the child." c. "The goal of palliative care is to place the child in a hospice setting at the end of life." d. "The goal of palliative care is to act as the liaison between the family, child, and other health care professionals."

ANS: B One of the goals of palliative care is to provide pain and symptom management.

36. A nurse is planning palliative care for a child with severe pain. Which should the nurse expect to be prescribed for pain relief? a. Opioids as needed b. Opioids on a regular schedule c. Distraction and relaxation techniques d. Nonsteroidal anti-inflammatory drugs

ANS: B Pain medications, for children in palliative care, should be given on a regular schedule, and extra doses for breakthrough pain should be available to maintain comfort. Opioid drugs such as morphine should be given for severe pain, and the dose should be increased as necessary to maintain optimal pain relief. Techniques such as distraction, relaxation techniques, and guided imagery should be combined with drug therapy to provide the child and family strategies to control pain. Nonsteroidal anti-inflammatory drugs are not sufficient to manage severe pain for children in palliative care.

32. The nurse should teach parents of a preschool child with type 1 diabetes that which can raise the blood glucose level? a. Exercise b. Steroids c. Decreased food intake d. Lantus insulin

ANS: B Parents should understand how to adjust food, activity, and insulin at the time of illness or when the child is treated for an illness with a medication known to raise the blood glucose level (e.g., steroids). Exercise, insulin, and decreased food intake can cause hypoglycemia.

12. The feeling of guilt that the child "caused" the disability or illness is especially critical in which child? a. Toddler b. Preschooler c. School-age child d. Adolescent

ANS: B Preschoolers are most likely to be affected by feelings of guilt that they caused the illness or disability or are being punished for wrongdoings. Toddlers are focused on establishing their autonomy. The illness will foster dependence. The school-age child will have limited opportunities for achievement and may not be able to understand limitations. Adolescents face the task of incorporating their disabilities into their changing self-concept.

17. The nurse should suspect a hearing impairment in an infant who demonstrates which behavior? a. Absence of the Moro reflex b. Absence of babbling by age 7 months c. Lack of eye contact when being spoken to d. Lack of gesturing to indicate wants after age 15 months

ANS: B The absence of babbling or inflections in voice by age 7 months is an indication of hearing difficulties. The absence of the Moro reflex and eye contact when being spoken to does not indicate a hearing impairment. The child with hearing impairment uses gestures rather than vocalizations to express desires at this age.

25. A 5-year-old boy is being prepared for surgery to remove a brain tumor. Nursing actions should be based on which statement? a. Removal of tumor will stop the various symptoms. b. Usually the postoperative dressing covers the entire scalp. c. He is not old enough to be concerned about his head being shaved. d. He is not old enough to understand the significance of the brain.

ANS: B The child should be told what he will look and feel like after surgery. This includes the size of the dressing. The nurse can demonstrate on a doll the expected size and shape of the dressing. Some of the symptoms may be alleviated by the removal of the tumor, but postsurgical headaches and cerebellar symptoms such as ataxia may be aggravated. Children should be prepared for the loss of their hair, and it should be removed in a sensitive, positive manner if the child is awake. Children at this age have poorly defined body boundaries and little knowledge of internal organs. Intrusive experiences are frightening, especially those that disrupt the integrity of the skin.

33. A nurse is reviewing the laboratory results on a school-age child with hypoparathyroidism. Which results are consistent with this condition? a. Decreased serum phosphorus b. Decreased serum calcium c. Increased serum glucose d. Decreased serum cortisol level

ANS: B The diagnosis of hypoparathyroidism is made on the basis of clinical manifestations associated with decreased serum calcium and increased serum phosphorus. A decreased serum phosphorus would be seen in hyperparathyroidism, elevated glucose in diabetes, and a decreased serum cortisol level in adrenocortical insufficiency (Addison disease).

5. Which syndrome involves a common sex chromosome defect? a. Down b. Turner c. Marfan d. Hemophilia

ANS: B Turner syndrome is caused by an absence of one of the X chromosomes. Down syndrome is caused by trisomy 21, three copies rather than two copies of chromosome 21. Marfan syndrome is a connective tissue disorder inherited in an autosomal dominant pattern. Hemophilia is a disorder of blood coagulation inherited in an X-linked recessive pattern.

1. The nurse case manager is planning a care conference about a young child who has complex health care needs and will soon be discharged home. Who should the nurse invite to the conference? a. Family and nursing staff b. Social worker, nursing staff, and primary care physician c. Family and key health professionals involved in child's care d. Primary care physician and key health professionals involved in the child's care

ANS: C A multidisciplinary conference is necessary for coordination of care for children with complex health needs. The family is included, along with key health professionals who are involved in the child's care. The nursing staff can address the child's nursing care needs with the family, but other involved disciplines must be included. The family must be included in the discharge conferences, which allows them to determine what education they will require and the resources needed at home. A member of the nursing staff must be included to review the child's nursing needs.

39. A child has been seizure-free for 2 years. A father asks the nurse how much longer the child will need to take the antiseizure medications. The nurse includes which intervention in the response? a. Medications can be discontinued at this time. b. The child will need to take the drugs for 5 years after the last seizure. c. A step-wise approach will be used to reduce the dosage gradually. d. Seizure disorders are a lifelong problem. Medications cannot be discontinued.

ANS: C A predesigned protocol is used to wean a child gradually off antiseizure medications, usually when the child is seizure-free for 2 years and has a normal electroencephalogram (EEG). Medications must be gradually reduced to minimize the recurrence of seizures. Seizure medications can be safely discontinued. The risk of recurrence is greatest within the first year.

23. A parent asks the nurse why self-monitoring of blood glucose is being recommended for her child with diabetes. The nurse should base the explanation on which knowledge? a. It is a less expensive method of testing. b. It is not as accurate as laboratory testing. c. Children are better able to manage the diabetes. d. Parents are better able to manage the disease.

ANS: C Blood glucose self-management has improved diabetes management and can be used successfully by children from the time of diagnosis. Insulin dosages can be adjusted based on blood glucose results. Blood glucose monitoring is more expensive but provides improved management. It is as accurate as equivalent testing done in laboratories. The ability to self-test allows the child to balance diet, exercise, and insulin. The parents are partners in the process, but the child should be taught how to manage the disease.

24. At what age do most children have an adult concept of death as being inevitable, universal, and irreversible? a. 4 to 5 years b. 6 to 8 years c. 9 to 11 years d. 12 to 16 years

ANS: C By age 9 to 11 years, children have an adult concept of death. They realize that it is inevitable, universal, and irreversible.

12. The nurse is teaching the parents of a child who is receiving methimazole (Tapazole) for the treatment of hyperthyroidism (Graves disease). Which statement made by the parent indicates a correct understanding of the teaching? a. "I would expect my child to gain weight while taking this medication." b. "I would expect my child to experience episodes of ear pain while taking this medication." c. "If my child develops a sore throat and fever, I should contact the physician immediately." d. "If my child develops the stomach flu, my child will need to be hospitalized."

ANS: C Children being treated with Tapazole must be carefully monitored for the side effects of the medication. Parents must be alerted that sore throat and fever accompany the grave complication of leukopenia. These symptoms should be immediately reported. Weight gain, episodes of ear pain, and concern for hospitalization with the stomach flu are not concerns related to taking Tapazole.

4. Which describes moral development in younger school-age children? a. The standards of behavior now come from within themselves. b. They do not yet experience a sense of guilt when they misbehave. c. They know the rules and behaviors expected of them but do not understand the reasons behind them. d. They no longer interpret accidents and misfortunes as punishment for misdeeds.

ANS: C Children who are ages 6 and 7 years know the rules and behaviors expected of them but do not understand the reasons for these rules and behaviors. Young children do not believe that standards of behavior come from within themselves, but that rules are established and set down by others. Younger school-age children learn standards for acceptable behavior, act according to these standards, and feel guilty when they violate them. Misfortunes and accidents are viewed as punishment for bad acts.

34. Which type of seizure involves both hemispheres of the brain? a. Focal b. Partial c. Generalized d. Acquired

ANS: C Clinical observations of generalized seizures indicate that the initial involvement is from both hemispheres. Focal seizures may arise from any area of the cerebral cortex, but the frontal, temporal, and parietal lobes are most commonly affected. Partial seizures are caused by abnormal electric discharges from epileptogenic foci limited to a circumscribed region of the cerebral cortex. A seizure disorder that is acquired is a result of a brain injury from a variety of factors; it does not specify the type of seizure.

7. When caring for a newborn with Down syndrome, the nurse should be aware that the most common congenital anomaly associated with Down syndrome is: a. hypospadias. b. pyloric stenosis. c. congenital heart disease. d. congenital hip dysplasia.

ANS: C Congenital heart malformations, primarily septal defects, are the most common congenital anomaly in Down syndrome. Hypospadias, pyloric stenosis, and congenital hip dysplasia are not frequent congenital anomalies associated with Down syndrome.

42. The nurse is monitoring a 7-year-old child post-surgical resection of an infratentorial brain tumor. Which vital sign findings indicate Cushing's triad? a. Increased temperature, tachycardia, tachypnea b. Decreased temperature, bradycardia, bradypnea c. Bradycardia, hypertension, irregular respirations d. Bradycardia, hypotension, tachypnea

ANS: C Cushing's triad is a hallmark sign of increased intracranial pressure (ICP). The triad includes bradycardia, hypertension, and irregular respirations. Increased or decreased temperature is not a sign of Cushing's triad.

3. A child has been diagnosed with enuresis. TCA imipramine (Tofranil) has been prescribed for the child. The nurse understands that this medication is in which category? a. Antidepressant b. Antidiuretic c. Antispasmodic d. Analgesic

ANS: C Drug therapy is increasingly being prescribed to treat enuresis. Three types of drugs are used: tricyclic antidepressants (TCAs), antidiuretics, and antispasmodics. The selection depends on the interpretation of the cause. The drug used most frequently is the TCA imipramine (Tofranil), which exerts an anticholinergic action in the bladder to inhibit urination. Tofranil is in the antispasmodic category. Analgesics are not used to treat enuresis.

9. The nurse is admitting a toddler with the diagnosis of juvenile hypothyroidism. Which is a common clinical manifestation of this disorder? a. Insomnia b. Diarrhea c. Dry skin d. Accelerated growth

ANS: C Dry skin, mental decline, and myxedematous skin changes are associated with juvenile hypothyroidism. Children with hypothyroidism are usually sleepy. Constipation is associated with hypothyroidism. Decelerated growth is common in juvenile hypothyroidism.

19. A nurse is teaching parents of kindergarten children general guidelines to assist their children in school. Which statement by the parents indicates they understand the teaching? a. "We will only meet with the teacher if problems occur." b. "We will discourage hobbies so our child focuses on school work." c. "We will plan a trip to the library as often as possible." d. "We will expect our child to make all As in school."

ANS: C General guidelines for parents to help their child in school include sharing an interest in reading. The library should be used frequently and books the child is reading should be discussed. Hobbies should be encouraged. The parents should not expect all As. They should focus on growth more than grades.

6. Which nursing intervention is especially helpful in assessing parental guilt when a disability or chronic illness is diagnosed? a. Ask the parents if they feel guilty. b. Discuss guilt only after the parents mention it. c. Discuss the meaning of the parents' religious and cultural background. d. Observe for signs of overprotectiveness.

ANS: C Guilt may be associated with cultural or religious beliefs. Some parents are convinced that they are being punished for some previous misdeed. Others may see the disorder as a sacrifice sent by God to test their religious beliefs. The nurse can help the parents explore their religious beliefs. The parents may not be able to identify the feelings of guilt. It would be appropriate for the nurse to explore their adjustment responses. Overprotectiveness is a parental response during the adjustment phase. The parents fear letting the child achieve any new skill and avoid all discipline.

24. The most common clinical manifestation(s) of brain tumors in children is/are: a. irritability. b. seizures. c. headaches and vomiting. d. fever and poor fine motor control.

ANS: C Headaches, especially on awakening, and vomiting that is not related to feeding are the most common clinical manifestation(s) of brain tumors in children. Irritability, seizures, and fever and poor fine motor control are clinical manifestations of brain tumors, but headaches and vomiting are the most common.

35. Which is the initial clinical manifestation of generalized seizures? a. Being confused b. Feeling frightened c. Losing consciousness d. Seeing flashing lights

ANS: C Loss of consciousness is a frequent occurrence in generalized seizures and is the initial clinical manifestation. Being confused, feeling frightened, and seeing flashing lights are clinical manifestations of a complex partial seizure.

1. A young child has an intelligence quotient (IQ) of 45. The nurse should document this finding as: a. within the lower limits of the range of normal intelligence. b. mild cognitive impairment but educable. c. moderate cognitive impairment but trainable. d. severe cognitive impairment and completely dependent on others for care.

ANS: C Moderate cognitively impairment IQs range between 35 and 55. The lower limit of normal intelligence is approximately 70. Individuals with IQs of 50 to 70 are considered to have mild cognitive impairment but educable. An IQ of 20 to 40 results in severe cognitive impairment.

1. The nurse has documented that a child's level of consciousness is obtunded. Which describes this level of consciousness? a. Slow response to vigorous and repeated stimulation b. Impaired decision making c. Arousable with stimulation d. Confusion regarding time and place

ANS: C Obtunded describes a level of consciousness in which the child is arousable with stimulation. Stupor is a state in which the child remains in a deep sleep, responsive only to vigorous and repeated stimulation. Confusion is impaired decision making. Disorientation is confusion regarding time and place.

19. Which is characteristic of the immune-mediated type 1 diabetes mellitus? a. Ketoacidosis is infrequent. b. Onset is gradual. c. Age at onset is usually younger than 20 years. d. Oral agents are often effective for treatment.

ANS: C The immune-mediated type 1 diabetes mellitus typically has its onset in children or young adults. Infrequent ketoacidosis, gradual onset, and effectiveness of oral agents for treatment are more consistent with type 2 diabetes.

26. At what developmental period do children have the most difficulty coping with death, particularly if it is their own? a. Toddlerhood b. Preschool c. School-age d. Adolescence

ANS: D Adolescents, because of their mature understanding of death, remnants of guilt and shame, and issues with deviations from normal, have the most difficulty coping with death.

24. Which is descriptive of bulimia during adolescence? a. Strong sense of control over eating behavior b. Feelings of elation after the binge-purge cycle c. Profound lack of awareness that the eating pattern is abnormal d. Weight that can be normal, slightly above normal, or below normal

ANS: D Individuals with bulimia are of normal or more commonly slightly above normal weight. Those who also restrict their intake can become severely underweight. The adolescent has a lack of control over eating during the episode. Patients with bulimia commonly have self-deprecating thoughts and a depressed mood after binge-purge cycles; they are also aware that the eating pattern is abnormal but are unable to stop.

15. The nurse should recommend medical attention if a child with a slight head injury experiences: a. sleepiness. b. vomiting, even once. c. headache, even if slight. d. confusion or abnormal behavior.

ANS: D Medical attention should be sought if the child exhibits confusion or abnormal behavior, loses consciousness, has amnesia, has fluid leaking from the nose or ears, complains of blurred vision, or has an unsteady gait. Sleepiness alone does not require evaluation. If the child is difficult to arouse from sleep, medical attention should be obtained. Vomiting more than three times requires medical attention. Severe or worsening headache or one that interferes with sleep should be evaluated.

5. The nurse is caring for a child with severe head trauma after a car accident. Which is an ominous sign that often precedes death? a. Papilledema b. Delirium c. Doll's head maneuver d. Periodic and irregular breathing

ANS: D Periodic or irregular breathing is an ominous sign of brainstem (especially medullary) dysfunction that often precedes complete apnea. Papilledema is edema and inflammation of optic nerve. It is commonly a sign of increased ICP. Delirium is a state of mental confusion and excitement marked by disorientation for time and place. The doll's head maneuver is a test for brainstem or oculomotor nerve dysfunction.

22. Type 1 diabetes mellitus is suspected in an adolescent. Which clinical manifestation may be present? a. Moist skin b. Weight gain c. Fluid overload d. Poor wound healing

ANS: D Poor wound healing may be present in an individual with type 1 diabetes mellitus. Dry skin, weight loss, and dehydration are clinical manifestations of type 1 diabetes mellitus.

29. An adolescent gets hit in the eye during a fight. The school nurse, using a flashlight, notes the presence of gross hyphema (hemorrhage into anterior chamber). The nurse should: a. apply a Fox shield. b. instruct the adolescent to apply ice for 24 hours. c. have adolescent rest with eye closed and ice applied. d. notify parents that adolescent needs to see an ophthalmologist.

ANS: D The parents should be notified that the adolescent needs to see an ophthalmologist as soon as possible. Applying a Fox shield, instructing the adolescent to apply ice for 24 hours, and having the adolescent rest with eye closed and ice applied may cause further damage. Referral to an ophthalmologist is indicated.

23. A preschooler is found digging up a pet bird that was recently buried after it died. The best explanation for this behavior is that: a. he has a morbid preoccupation with death. b. he is looking to see whether a ghost took it away. c. the loss is not yet resolved, and professional counseling is needed. d. reassurance is needed that the pet has not gone somewhere else.

ANS: D The preschooler can recognize that the pet has died but has difficulties with the permanence. Digging up the bird gives reassurance that the bird is still present.

10. Teasing can be common during the school-age years. The nurse should recognize that which applies to teasing? a. Can have a lasting effect on children b. Is not a significant threat to self-concept c. Is rarely based on anything that is concrete d. Is usually ignored by the child who is being teased

NS: A Teasing in this age group is common and can have a long-lasting effect. Increasing awareness of differences, especially when accompanied by unkind comments and taunts from others, may make a child feel inferior and undesirable. Physical impairments such as hearing or visual defects, ears that "stick out," or birth marks assume great importance.

11. Exophthalmos (protruding eyeballs) may occur in children with: a. hypothyroidism. b. hyperthyroidism. c. hypoparathyroidism. d. hyperparathyroidism.

ANS: B Exophthalmos is a clinical manifestation of hyperthyroidism. Hypothyroidism, hypoparathyroidism, and hyperparathyroidism are not associated with exophthalmos.

33. The nurse is providing support to a family who is experiencing anticipatory grief related to their child's imminent death. Which of the following is an appropriate nursing intervention? a. Be available to family. b. Attempt to "lighten the mood." c. Suggest activities to cheer up the family. d. Discourage crying until actual time of death.

ANS: A One of the most important nursing interventions of death is the availability of the nurse for the family.

9. A common parental reaction to a child with special needs is parental overprotection. Parental behavior suggestive of this includes: a. giving inconsistent discipline. b. providing consistent, strict discipline. c. forcing child to help self, even when not capable. d. encouraging social and educational activities not appropriate to child's level of capability.

ANS: A Parental overprotection is manifested by the parents' fear of letting the child achieve any new skill; they allow the child to avoid all discipline and cater to every desire to prevent frustration. Overprotective parents: do not set limits and or institute discipline; prefer to remain in the role of total caregiver; do not allow the child to perform self-care; and do not encourage the child to participate in social and educational activities.

1. The clinic nurse is reviewing hemoglobin A1c levels on several children with type 1 diabetes. Hemoglobin A1c levels of less than _____% is a goal for children with type 1 diabetes. (Record your answer in a whole number.)

ANS: 7 The measurement of glycosylated hemoglobin (hemoglobin A1c) levels is a satisfactory method for assessing control of the diabetes. As red blood cells circulate in the bloodstream, glucose molecules gradually attach to the hemoglobin A molecules and remain there for the lifetime of the red blood cell, approximately 120 days. The attachment is not reversible; therefore, this glycosylated hemoglobin reflects the average blood glucose levels over the previous 2 to 3 months. The test is a satisfactory method for assessing control, detecting incorrect testing, monitoring the effectiveness of changes in treatment, defining patients' goals, and detecting nonadherence. Hemoglobin A1c levels of less than 7% are a well-established goal at most care centers.

15. The nurse is providing support to parents at the time their child is diagnosed with chronic disabilities. The nurse notices that the parents keep asking the same questions. What is the nurse's best intervention? a. Patiently continue to answer questions. b. Kindly refer them to someone else for answering their questions. c. Recognize that some parents cannot understand explanations. d. Suggest that they ask their questions when they are not upset.

ANS: A Diagnosis is one of the anticipated stress points for parents. The parents may not hear or remember all that is said to them. The nurse should continue to provide the kind of information they desire. This is a particularly stressful time for the parents; the nurse can play a key role in providing necessary information. Parents should be provided with oral and written information. The nurse needs to work with the family to ensure understanding of the information. The parents require information at the time of diagnosis. Other questions will arise as they adjust to the information.

18. The parents of a child born with disabilities ask the nurse for advice about discipline. The nurse's response should be based on knowledge that discipline is: a. essential for the child. b. too difficult to implement with a special-needs child. c. not needed unless child becomes problematic. d. best achieved with punishment for misbehavior.

ANS: A Discipline is essential for the child. It provides boundaries on which to test out their behavior and teaches them socially acceptable behaviors. The nurse should teach the parents ways to manage the child's behavior before it becomes problematic. Punishment is not effective in managing behavior.

25. A nurse should suspect possible visual impairment in a child who displays which characteristic? a. Excessive rubbing of the eyes b. Rapid lateral movement of the eyes c. Delay in speech development d. Lack of interest in casual conversation with peers

ANS: A Excessive rubbing of the eyes is a clinical manifestation of visual impairment. Rapid lateral movement of the eyes, delay in speech development, and lack of interest in casual conversation with peers are not associated with visual impairment.

36. Which is the most commonly used method in completed suicides? a. Firearms b. Drug overdose c. Self-inflected laceration d. Carbon monoxide poisoning

ANS: A Firearms are the most commonly used instruments in completed suicides among both males and females. For adolescent boys, firearms are followed by hanging and overdose. For adolescent females, overdose and strangulation are the next most common means of completed suicide. The most common method of suicide attempt is overdose or ingestion of potentially toxic substances such as drugs. The second most common method of suicide attempt is self-inflicted laceration. Carbon monoxide poisoning is not one of the more frequent forms of suicide completion.

10. Which drug should the nurse expect to administer to a preschool child who has increased intracranial pressure (ICP) resulting from cerebral edema? a. Mannitol (Osmitrol) b. Epinephrine hydrochloride (Adrenalin) c. Atropine sulfate (Atropine) d. Sodium bicarbonate (Sodium bicarbonate)

ANS: A For increased ICP, mannitol, an osmotic diuretic, administered intravenously, is the drug used most frequently for rapid reduction. Epinephrine hydrochloride, atropine sulfate, and sodium bicarbonate are not used to decrease ICP.

18. Which statement is correct about childhood obesity? a. Heredity is an important factor in the development of obesity. b. Childhood obesity in the United States is decreasing. c. Childhood obesity is the result of inactivity. d. Childhood obesity can be attributed to an underlying disease in most cases.

ANS: A Heredity is an important fact that contributes to obesity. Identical twins reared apart tend to resemble their biologic parents to a greater extent than their adoptive parents. It is difficult to distinguish between hereditary and environmental factors. The number of overweight children is increasing in the United States. Inactivity is related to childhood obesity, but it is not the only component. Underlying diseases such as hypothyroidism and hyperinsulinism account for only a small number of cases of childhood obesity.

23. Which term refers to the ability to see objects clearly at close range but not at a distance? a. Myopia b. Amblyopia c. Cataract d. Glaucoma

ANS: A Myopia, or nearsightedness, refers to the ability to see objects clearly at close range but not a distance. Amblyopia, or lazy eye, is reduced visual acuity in one eye. A cataract is opacity of the lens of the eye. Glaucoma is a group of eye diseases characterized by increased intraocular pressure.

27. The nurse is teaching the parents of a child recently diagnosed with ADHD who has been prescribed methylphenidate (Ritalin). Which should the nurse include in teaching about the side effects of methylphenidate? a. "Your child may experience a sense of nervousness." b. "You may see an increase in your child's appetite." c. "Your child may experience daytime sleepiness." d. "You may see a decrease in your child's blood pressure."

ANS: A Nervousness is one of the common side effects of Ritalin. Decreased appetite with subsequent weight loss, insomnia, and increased blood pressure are other common side effects.

26. The nurse is teaching nursing students about childhood nervous system tumors. Which best describes a neuroblastoma? a. Diagnosis is usually made after metastasis occurs. b. Early diagnosis is usually possible because of the obvious clinical manifestations. c. It is the most common brain tumor in young children. d. It is the most common benign tumor in young children.

ANS: A Neuroblastoma is a silent tumor with few symptoms. In more than 70% of cases, diagnosis is made after metastasis occurs, with the first signs caused by involvement in the nonprimary site. In only 30% of cases is diagnosis made before metastasis. Neuroblastomas are the most common malignant extracranial solid tumors in children. The majority of tumors develop in the adrenal glands or the retroperitoneal sympathetic chain. They are not benign but metastasize.

5. Which statement characterizes moral development in the older school-age child? a. They are able to judge an act by the intentions that prompted it rather than just by the consequences. b. Rules and judgments become more absolute and authoritarian. c. They view rule violations in an isolated context. d. They know the rules but cannot understand the reasons behind them.

ANS: A Older school-age children are able to judge an act by the intentions that prompted the behavior rather than just by the consequences. Rules and judgments become less absolute and authoritarian. Rule violation is likely to be viewed in relation to the total context in which it appears. The situation and the morality of the rule itself influence reactions.

34. A child with autism spectrum disorder (ASD) is admitted to the hospital with pneumonia. The nurse should plan which priority intervention when caring for the child? a. Maintain a structured routine and keep stimulation to a minimum. b. Place child in a room with a roommate of the same age. c. Maintain frequent touch and eye contact with the child. d. Take the child frequently to the playroom to play with other children.

ANS: A Providing a structured routine for the child to follow is a key in the management of ASD. Decreasing stimulation by using a private room, avoiding extraneous auditory and visual distractions, and encouraging the parents to bring in possessions the child is attached to may lessen the disruptiveness of hospitalization. Because physical contact often upsets these children, minimum holding and eye contact may be necessary to avoid behavioral outbursts. Children with ASD need to be introduced slowly to new situations, with visits with staff caregivers kept short whenever possible. The playroom would be too overwhelming with new people and situations and should not be a priority of care.

9. Which is the priority nursing intervention for an unconscious child after a fall? a. Establish adequate airway. b. Perform neurologic assessment. c. Monitor intracranial pressure. d. Determine whether a neck injury is present.

ANS: A Respiratory effectiveness is the primary concern in the care of the unconscious child. Establishment of an adequate airway is always the first priority. A neurologic assessment and determination of whether a neck injury is present will be performed after breathing and circulation are stabilized. Intracranial, not intercranial, pressure is monitored if indicated after airway, breathing, and circulation are maintained.

38. An adolescent girl tells the nurse that she is very suicidal. The nurse asks her whether she has a specific plan. Asking this should be considered: a. an appropriate part of the assessment. b. not a critical part of the assessment. c. suggesting that the adolescent needs a plan. d. encouraging the adolescent to devise a plan.

ANS: A Routine health assessments of adolescents should include questions that assess the presence of suicidal ideation or intent. Questions such as, "Have you ever developed a plan to hurt yourself or kill yourself?" should be part of that assessment. Adolescents who express suicidal feelings and have a specific plan are at particular risk and require further assessment and constant monitoring. The information about having a plan is an essential part of the assessment and greatly affects the treatment plan.

4. Which should be the major consideration when selecting toys for a child who is cognitively impaired? a. Safety b. Age appropriateness c. Ability to provide exercise d. Ability to teach useful skills

ANS: A Safety is the primary concern in selecting recreational and exercise activities for all children. This is especially true for children who are cognitively impaired. Age appropriateness, the ability to provide exercise, and the ability to teach useful skills should all be considered in the selection of toys, but safety is of paramount importance.

17. An adolescent boy is brought to the emergency department after a motorcycle accident. His respirations are deep, periodic, and gasping. There are extreme fluctuations in blood pressure. Pupils are dilated and fixed. The nurse should suspect which type of head injury? a. Brainstem b. Skull fracture c. Subdural hemorrhage d. Epidural hemorrhage

ANS: A Signs of brainstem injury include deep, rapid, periodic or intermittent, and gasping respirations. Wide fluctuations or noticeable slowing of the pulse, widening pulse pressure, or extreme fluctuations in blood pressure are consistent with a brainstem injury. Skull fracture, subdural hemorrhage, and epidural hemorrhage are not consistent with brainstem injuries.

12. A 9-year-old girl often comes to the school nurse complaining of stomach pains. Her teacher says she is completing her school work satisfactorily but lately has been somewhat aggressive and stubborn in the classroom. The school nurse should recognize this as: a. signs of stress. b. developmental delay. c. physical problem causing emotional stress. d. lack of adjustment to school environment.

ANS: A Signs of stress include stomach pains or headache, sleep problems, bed-wetting, changes in eating habits, aggressive or stubborn behavior, reluctance to participate, or regression to early behaviors. This child is exhibiting signs of stress.

37. Which is the most significant factor in distinguishing those who commit suicide from those who make suicidal attempts or threats? a. Social isolation b. Level of stress c. Degree of depression d. Desire to punish others

ANS: A Social isolation is a significant factor in distinguishing adolescents who will kill themselves from those who will not. It is also more characteristic of those who complete suicide than of those who make attempts or threats. Level of stress, degree of depression, and desire to punish others are contributing factors in suicide, but they are not the most significant factor in distinguishing those who complete suicide from those who attempt suicide.

38. A 10-year-old child, without a history of previous seizures, experiences a tonic-clonic seizure at school. Breathing is not impaired, but some postictal confusion occurs. The most appropriate initial action by the school nurse is to: a. stay with child and have someone call emergency medical service (EMS). b. notify parent and regular practitioner. c. notify parent that child should go home. d. stay with child, offering calm reassurance.

ANS: A The EMS should be called to transport the child because this is the child's first seizure. Because this is the first seizure, evaluation should be performed as soon as possible. The nurse should stay with the child while someone else notifies the EMS.

20. A nurse is caring for a hearing-impaired child who lip reads. The nurse should plan which intervention to facilitate lip reading? a. Speak at an even rate. b. Exaggerate pronunciation of words. c. Avoid using facial expressions. d. Repeat in exactly the same way if child does not understand.

ANS: A The child should be helped to learn and understand how to read lips by speaking at an even rate. It interferes with the child's comprehension of the spoken word to exaggerate pronunciation of words, to avoid using facial expressions, and to repeat in exactly the same way if the child does not understand.

21. Parents of a twelve-year-old child ask the clinic nurse, "How many hours of sleep should our child get?' The nurse should respond that 12-year-old children need how many hours of sleep at night? a. 8 b. 9 c. 10 d. 11

ANS: B School-age children usually do not require naps, but they do need to sleep approximately 11 hours at age 5 years and 9 hours at age 12 years each night.

21. A school-age child has sustained a head injury and multiple fractures after being thrown from a horse. The child's level of consciousness is variable. The parents tell the nurse that they think their child is in pain because of periodic crying and restlessness. The most appropriate nursing action is to: a. discuss with parents the child's previous experiences with pain. b. discuss with practitioner what analgesia can be safely administered. c. explain that analgesia is contraindicated with a head injury. d. explain that analgesia is unnecessary when child is not fully awake and alert.

ANS: B A key nursing role is to provide sedation and analgesia for the child. Consultation with the appropriate practitioner is necessary to avoid conflict between the necessity to monitor the child's neurologic status and the promotion of comfort and relief of anxiety. Information on the child's previous experiences with pain should be obtained as part of the assessment, but because of the severity of injury, analgesia should be provided as soon as possible. Analgesia can be safely used in individuals who have sustained head injuries and can decrease anxiety and resultant increased ICP.

31. At the time of a child's death, the nurse tells his mother, "We will miss him so much." The best interpretation of this is that the nurse is: a. pretending to be experiencing grief. b. expressing personal feelings of loss. c. denying the mother's sense of loss. d. talking when listening would be better.

ANS: B A patient's death is one of the most stressful aspects of critical care or oncology nursing. Nurses experience reactions similar to those of family members because of their involvement with the child and family during the illness. Nurses often have feelings of personal loss when a patient dies.

35. A school nurse is conducting a class with adolescents on suicide. Which true statement about suicide should the nurse include in the teaching session? a. A sense of hopelessness and despair are a normal part of adolescence. b. Gay and lesbian adolescents are at a particularly high risk for suicide. c. Problem-solving skills are of limited value to the suicidal adolescent. d. Previous suicide attempts are not an indication of risk for completed suicides.

ANS: B A significant number of teenage suicides occur among homosexual youths. Gay and lesbian adolescents who live in families or communities that do not accept homosexuality are likely to suffer low self-esteem, self-loathing, depression, and hopelessness as a result of a lack of acceptance from their family or community. At-risk teenagers include those who are depressed, have poor problem-solving skills, or use drugs and alcohol. History of previous suicide attempt is a serious indicator for possible suicide completion in the future.

14. Which statement best describes a subdural hematoma? a. Bleeding occurs between the dura and the skull. b. Bleeding occurs between the dura and the cerebrum. c. Bleeding is generally arterial, and brain compression occurs rapidly. d. The hematoma commonly occurs in the parietotemporal region.

ANS: B A subdural hematoma is bleeding that occurs between the dura and the cerebrum as a result of a rupture of cortical veins that bridge the subdural space. An epidural hemorrhage occurs between the dura and the skull, is usually arterial with rapid brain concussion, and occurs most often in the parietotemporal region.

35. Parents are asking about an early intervention program for their child who has special needs. The nurse relates that this program is for which age of child? a. Birth to 1 year of age b. Birth to 3 years of age c. Ages 1 to 4 d. Ages 4 and 5

ANS: B A variety of supplemental programs have been designed in the school system to accommodate special needs, both at school age and younger, through early intervention, which consists of any sustained and systematic effort to assist children from birth to age 3 years with disabilities and those who are developmentally vulnerable.

26. Which is descriptive of attention deficit hyperactivity disorder (ADHD)? a. Manifestations exhibited are so bizarre that the diagnosis is fairly easy. b. Manifestations affect every aspect of the child's life but are most obvious in the classroom. c. Learning disabilities associated with ADHD eventually disappear when adulthood is reached. d. Diagnosis of ADHD requires that all manifestations of the disorder be present.

ANS: B ADHD affects every aspect of the child's life, but the disruption is most obvious in the classroom. The behaviors exhibited by the child with ADHD are not unusual aspects of behavior. The difference lies in the quality of motor activity and developmentally inappropriate inattention, impulsivity, and hyperactivity that the child displays. Some children experience decreased symptoms during late adolescence and adulthood, but a significant number carry their symptoms into adulthood. Any given child will not have every symptom of the condition. The manifestations may be numerous or few, mild or severe, and will vary with the child's developmental level.

19. Kelly, an 8-year-old girl, will soon be able to return to school after an injury that resulted in several severe, chronic disabilities. Which is the most appropriate action by the school nurse? a. Recommend that the child's parents attend school at first to prevent teasing. b. Prepare the child's classmates and teachers for changes they can expect. c. Refer the child to a school where the children have chronic disabilities similar to hers. d. Discuss with the child and her parents the fact that her classmates will not accept her as they did before.

ANS: B Attendance at school is an important part of normalization for Kelly. The school nurse should prepare teachers and classmates about her condition, abilities, and special needs. A visit by the parents can be helpful, but unless the classmates are prepared for the changes, it alone will not prevent teasing. Kelly's school experience should be normalized as much as possible. Children need the opportunity to interact with healthy peers, as well as to engage in activities with groups or clubs composed of similarly affected persons. Children with special needs are encouraged to maintain and reestablish relationships with peers and to participate according to their capabilities.

40. An adolescent has been diagnosed with Chlamydia infection. Which medication should the nurse expect to be prescribed for this condition? a. Ceftriaxone (Rocephin) IM b. Azithromycin (Zithromax) PO c. Acyclovir (Zovirax) PO d. Penicillin G benzathine (Bicillin) IV

ANS: B Azithromycin is used to treat Chlamydia. The patient should be rescreened in 3 to 4 months. Ceftriaxone is used to treat gonorrhea, acyclovir is used to suppress genital herpes simplex virus, and penicillin G benzathine is used to treat syphilis.

22. Which best describes how preschoolers react to the death of a loved one? a. Preschooler is too young to have a concept of death. b. Preschooler may feel guilty and responsible for the death. c. Grief is acute but does not last long at this age. d. Grief is usually expressed in the same way in which the adults in the preschooler's life are expressing grief.

ANS: B Because of egocentricity, the preschooler may feel guilty and responsible for the death.

8. Mark, a 9-year-old with Down syndrome, is mainstreamed into a regular third-grade class for part of the school day. His mother asks the school nurse about programs, such as Cub Scouts, that he might join. The nurse's recommendation should be based on which statement? a. Programs like Cub Scouts are inappropriate for children who are mentally retarded. b. Children with Down syndrome have the same need for socialization as other children. c. Children with Down syndrome socialize better with children who have similar disabilities. d. Parents of children with Down syndrome encourage programs, such as scouting, because they deny that their children have disabilities.

ANS: B Children of all ages need peer relationships. Children with Down syndrome should have peer experiences similar to those of other children, such as group outings, Cub Scouts, and Special Olympics. Programs such as Cub Scouts can help children with cognitive impairment develop socialization skills. Although all children should have an opportunity to form a close relationship with someone of the same developmental level, it is appropriate for children with disabilities to develop relationships with children who do not have disabilities. The parents are acting as advocates for their child.

19. The psychological effects of being obese during adolescence include: a. sexual promiscuity. b. poor body image. c. feelings of contempt for thin peers. d. accurate body image but self-deprecating attitude.

ANS: B Common emotional consequences of obesity include poor body image, low self-esteem, social isolation, and feelings of depression and isolation. Sexual promiscuity, feelings of contempt for thin peers, and accurate body image but self-deprecating attitude are not usually associated with obesity.

17. A neonate born with ambiguous genitalia is diagnosed with congenital adrenogenital hyperplasia. Therapeutic management includes administration of: a. vitamin D. b. cortisone. c. stool softeners. d. calcium carbonate.

ANS: B Cortisone is administered to suppress the abnormally high secretions of adrenocorticotropic hormone (ACTH). This in turn inhibits the secretion of adrenocorticosteroid, which stems the progressive virilization. Vitamin D, stool softeners, and calcium carbonate have no role in the therapy of adrenogenital hyperplasia.

33. Parents of a child with Down syndrome ask the nurse about techniques for introducing solid food to their 8-month-old child's diet. The nurse should give the parents which priority instruction? a. It is too early to add solids; the parents should wait for 2 to 3 months. b. A small but long, straight-handled spoon should be used to push the food toward the back and side of the mouth. c. If the child thrusts the food out, the feeding should be stopped. d. Solids should be offered only three times a day.

ANS: B Down syndrome children have a protruding tongue which can interfere with feeding, especially of solid foods. Parents need to know that the tongue thrust is not an indication of refusal to feed but a physiologic response. Parents are advised to use a small but long, straight-handled spoon to push the food toward the back and side of the mouth. If food is thrust out, it should be re-fed. Six months is the time to introduce solid foods to a child, so waiting 2 to 3 months is inappropriate. Small frequent feedings should be initiated to prevent the child from tiring. Three times a day is too infrequent.

13. Which statement best describes fear in the school-age child? a. They are increasingly fearful for body safety. b. Most of the new fears that trouble them are related to school and family. c. They should be encouraged to hide their fears to prevent ridicule by peers. d. Those who have numerous fears need continuous protective behavior by parents to eliminate these fears.

ANS: B During the school-age years, children experience a wide variety of fears, but new fears relate predominantly to school and family. During the middle-school years, children become less fearful for body safety than they were as preschoolers. Parents and other persons involved with children should discuss children's fears with them individually or as a group activity. Sometimes school-age children hide their fears to avoid being teased. Hiding their fears does not end them and may lead to phobias.

11. Which intervention will encourage a sense of autonomy in a toddler with disabilities? a. Avoid separation from family during hospitalizations. b. Encourage independence in as many areas as possible. c. Expose child to pleasurable experiences as much as possible. d. Help parents learn special care needs of their child.

ANS: B Encouraging the toddler to be independent encourages a sense of autonomy. The child can be given choices about feeding, dressing, and diversional activities, which will provide a sense of control. Avoiding separation from family during hospitalizations, and helping parents learn special care needs of their child should be practiced as part of family-centered care. It does not necessarily foster autonomy. Exposing the child to pleasurable experiences, especially sensory ones, is a supportive intervention. It does not promote autonomy.

4. An adolescent is being seen in the clinic for evaluation of acromegaly. The nurse understands that which occurs with acromegaly? a. There is a lack of growth hormone (GH) being produced. b. There is excess growth hormone (GH) after closure of the epiphyseal plates. c. There is an excess of growth hormone (GH) before the closure of the epiphyseal plates. d. There is a lack of thyroid hormone being produced.

ANS: B Excess GH after closure of the epiphyseal plates results in acromegaly. A lack of growth hormone results in delayed growth or even dwarfism. Gigantism occurs when there is hypersecretion of GH before the closure of the epiphyseal plates. Cretinism is associated with hypothyroidism.

5. Families progress through various stages of reactions when a child is diagnosed with a chronic illness or disability. After the shock phase, a period of adjustment usually follows. This is often characterized by which of the following responses? a. Denial b. Guilt and anger c. Social reintegration d. Acceptance of child's limitations

ANS: B For most families, the adjustment phase is accompanied by several responses. Guilt, self-accusation, bitterness, and anger are common reactions. The initial diagnosis of a chronic illness or disability often is met with intense emotion, characterized by shock and denial. Social reintegration and acceptance of the child's limitations are the culmination of the adjustment process.

10. A newborn assessment shows separated sagittal suture, oblique palpebral fissures, depressed nasal bridge, protruding tongue, and transverse palmar creases. These findings are most suggestive of: a. microcephaly. b. Down syndrome. c. cerebral palsy. d. fragile X syndrome.

ANS: B These are characteristics associated with Down syndrome. The infant with microcephaly has a small head. Cerebral palsy is a diagnosis not usually made at birth. No characteristic physical signs are present. The infant with fragile X syndrome has increased head circumference; long, wide, and/or protruding ears; long, narrow face with prominent jaw; hypotonia; and high arched palate.

43. Which position should the nurse place a 10-year-old child after a large tumor was removed through a supratentorial craniotomy? a. On the inoperative side with the bed flat b. On the inoperative side with the head of bed elevated 20 to 30 degrees c. On the operative side with the bed flat and pillows behind the head d. On the operative side with the head of bed elevated 45 degrees

ANS: B If a large tumor was removed, the child is not placed on the operative side because the brain may suddenly shift to that cavity, causing trauma to the blood vessels, linings, and the brain itself. The child with an infratentorial procedure is usually positioned on either side with the bed flat. When a supratentorial craniotomy is performed, the head of bed is elevated 20 to 30 degrees with the child on either side or on the back. In a supratentorial craniotomy, the head elevation facilitates CSF drainage and decreases excessive blood flow to the brain to prevent hemorrhage. Pillows should be placed against the child's back, not head, to maintain the desired position.

1. The nurse is teaching a group of 10- to 12-year-old children about physical development during the school-age years. Which statement made by a participant, indicates the correct understanding of the teaching? a. "My body weight will be almost triple in the next few years." b. "I will grow an average of 2 inches per year from this point on." c. "There are not that many physical differences among school-age children." d. "I will have a gradual increase in fat, which may contribute to a heavier appearance."

ANS: B In middle childhood, growth in height and weight occurs at a slower pace. Between the ages of 6 and 12 years, children grow 2 inches per year. In middle childhood, children's weight will almost double; they gain 3 kg/year. At the end of middle childhood, girls grow taller and gain more weight than boys. Children take on a slimmer look with longer legs in middle childhood.

2. Which represents a common best practice in the provision of services to children with special needs? a. Care is now being focused on the child's chronologic age. b. Children with special needs are being integrated into regular classrooms. c. Children with special needs no longer have to be cared for by their families. d. Children with special needs are being separated into residential treatment facilities.

ANS: B Normalization refers to behaviors and interventions for the disabled to integrate into society by living life as persons without a disability would. For children, normalization includes attending school and being integrated into regular classrooms. This affords the child the advantages of learning with a wide group of peers. Care is necessarily focused on the child's developmental age. Home care by the family is considered best practice. The nurse can assist families by assessing social support systems, coping strategies, family cohesiveness, and family and community resources.

8. The nurse comes into the room of a child who was just diagnosed with a chronic disability. The child's parents begin to yell at the nurse about a variety of concerns. Which is the nurse's best response? a. "What is really wrong?" b. "Being angry is only natural." c. "Yelling at me will not change things." d. "I will come back when you settle down."

ANS: B Parental anger after the diagnosis of a child with a chronic disability is a common response. One of the most common targets for parental anger is members of the staff. The nurse should recognize the common response of anger to the diagnosis and allow the family to ventilate. "What is really wrong?"/"Yelling at me will not change things"/"I will come back when you settle down" will place the parents on the defensive and not facilitate communication.

3. Lindsey, age 5 years, will be starting kindergarten next month. She has cerebral palsy, and it has been determined that she needs to be in a special education classroom. Her parents are tearful when telling the nurse about this and state that they did not realize her disability was so severe. The best interpretation of this situation is that: a. this is a sign parents are in denial. b. this is a normal anticipated time of parental stress. c. the parents need to learn more about cerebral palsy. d. the parents are used to having expectations that are too high.

ANS: B Parenting a child with a chronic illness can be stressful for parents. There are anticipated times that parental stress increases. One of these identified times is when the child begins school. Nurses can help parents recognize and plan interventions to work through these stressful periods. The parents are not in denial; they are responding to the child's placement in school. The parents are not exhibiting signs of a knowledge deficit; this is their first interaction with the school system with this child.

32. A teen asks a nurse, "What is physical dependence in substance abuse?" Which is the correct response by the nurse? a. Problem that occurs in conjunction with addiction b. Involuntary physiologic response to drug c. Culturally defined use of drugs for purposes other than accepted medical purposes d. Voluntary behavior based on psychosocial needs

ANS: B Physical dependence is an involuntary response to the pharmacologic characteristics of drugs such as opioids or alcohol. A person can be physically dependent on a narcotic/drug without being addicted; for example, patients who use opioids to control pain need increasing doses to achieve the same effect. Dependence is a physiologic response; it is not culturally determined or subject to voluntary control.

9. A school nurse observes school-age children playing at recess. Which is descriptive of the play the nurse expects to observe? a. Individuality in play is better tolerated than at earlier ages. b. Knowing the rules of a game gives an important sense of belonging. c. They like to invent games, making up the rules as they go. d. Team play helps children learn the universal importance of competition and winning.

ANS: B Play involves increased physical skill, intellectual ability, and fantasy. Children form groups and cliques and develop a sense of belonging to a team or club. At this age, children begin to see the need for rules. Conformity and ritual permeate their play. Their games have fixed and unvarying rules, which may be bizarre and extraordinarily rigid. With team play, children learn about competition and the importance of winning, an attribute highly valued in the United States.

28. The nurse is caring for an 11-year-old boy who has recently been diagnosed with diabetes. Which should be included in the teaching plan for daily injections? a. The parents do not need to learn the procedure. b. He is old enough to give most of his own injections. c. Self-injections will be possible when he is closer to adolescence. d. He can learn about self-injections when he is able to reach all injection sites.

ANS: B School-age children are able to give their own injections. Parents should participate in learning and giving the insulin injections. He is already old enough to administer his own insulin. The child is able to use thighs, abdomen, part of the hip, and arm. Assistance can be obtained if other sites are used.

12. An adolescent tells the school nurse that she is pregnant. Her last menstrual period was 4 months ago. She has not received any medical care. She smokes but denies any other substance use. The priority nursing action is to: a. notify her parents. b. refer for prenatal care. c. explain the importance of not smoking. d. discuss dietary needs for adequate fetal growth.

ANS: B Teenage girls and their unborn children are at greater risk for complications during pregnancy and delivery. With improved therapies, the mortality for teenage pregnancy is decreasing, but the morbidity is high. A pregnant teenager needs careful assessment by the nurse to determine the level of social support available to her and possibly her partner. Guidance from the adults in her life would be invaluable, but confidentiality should be maintained. Although it is important to explain the importance of not smoking and to discuss dietary needs for adequate fetal growth, because of her potential for having a high-risk pregnancy, she will need a comprehensive prenatal program to minimize maternal-fetal complications.

5. Appropriate interventions to facilitate socialization of the cognitively impaired child include: a. providing age-appropriate toys and play activities. b. providing peer experiences, such as scouting, when older. c. avoiding exposure to strangers who may not understand cognitive development. d. emphasizing mastery of physical skills because they are delayed more often than verbal skills.

ANS: B The acquisition of social skills is a complex task. Children of all ages need peer relationships. Parents should enroll the child in preschool. When older, they should have peer experiences similar to those of other children such as group outings, Boy and Girl Scouts, and Special Olympics. It is important to provide age-appropriate toys and play activities, but peer interactions will facilitate social development. Parents should expose the child to strangers so that the child can practice social skills. Verbal skills are delayed more than physical skills.

8. A nasal spray of desmopressin acetate (DDAVP) is used to treat which disorder? a. Hypopituitarism b. Diabetes insipidus c. Acute adrenocortical insufficiency d. Syndrome of inappropriate antidiuretic hormone

ANS: B The drug of choice for the treatment of diabetes insipidus is DDAVP, which is a synthetic analogue of vasopressin. DDAVP is not used to treat hypopituitarism, acute adrenocortical insufficiency, or syndrome of inappropriate antidiuretic hormone.

2. The nurse has received report on four children. Which child should the nurse assess first? a. A school-age child in a coma with stable vital signs b. A preschool child with a head injury and decreasing level of consciousness c. An adolescent admitted after a motor vehicle accident is oriented to person and place d. A toddler in a persistent vegetative state with a low-grade fever

ANS: B The nurse should assess the child with a head injury and decreasing level of consciousness first (LOC). Assessment of LOC remains the earliest indicator of improvement or deterioration in neurologic status. The next child the nurse should assess is a toddler in a persistent vegetative state with a low-grade fever. The school-age child in a coma with stable vital signs and the adolescent admitted to the hospital who is oriented to his surroundings would be of least worry to the nurse.

27. The school nurse is caring for a child with a penetrating eye injury. Emergency treatment includes which intervention? a. Apply a regular eye patch. b. Apply a Fox shield to affected eye and any type of patch to the other eye. c. Apply ice until the physician is seen. d. Irrigate eye copiously with a sterile saline solution.

ANS: B The nurse's role in a penetrating eye injury is to prevent further injury to the eye. A Fox shield (if available) should be applied to the injured eye and a regular eye patch to the other eye to prevent bilateral movement. It may cause more damage to the eye to apply a regular eye patch, apply ice until the physician is seen, or irrigate the eye copiously with a sterile saline solution.

4. Approach behaviors are those coping mechanisms that result in a family's movement toward adjustment and resolution of the crisis of having a child with a chronic illness or disability. Which is considered an approach behavior? a. Is unable to adjust to a progression of the disease or condition b. Anticipates future problems and seeks guidance and answers c. Looks for new cures without a perspective toward possible benefit d. Fails to recognize seriousness of child's condition despite physical evidence

ANS: B The parents who anticipate future problems and seek guidance and answers are demonstrating approach behaviors. They are demonstrating positive actions in caring for their child. Being unable to adjust to a progression of the disease or condition, looking for new cures without a perspective toward possible benefit, and failing to recognize seriousness of child's condition despite physical evidence are avoidance behaviors. The parents are moving away from adjustment (and toward maladaptation) in the crisis of a child with chronic illness or disability.

6. The nurse is conducting a staff in-service on childhood endocrine disorders. Diabetes insipidus is a disorder of: a. anterior pituitary. b. posterior pituitary. c. adrenal cortex. d. adrenal medulla.

ANS: B The principal disorder of posterior pituitary hypofunction is diabetes insipidus. The anterior pituitary produces hormones such as GH, thyroid-stimulating hormone, adrenocorticotropic hormone, gonadotropin, prolactin, and melanocyte-stimulating hormone. The adrenal cortex produces aldosterone, sex hormones, and glucocorticoids. The adrenal medulla produces catecholamines.

4. The nurse is closely monitoring a child who is unconscious after a fall and notices that the child suddenly has a fixed and dilated pupil. The nurse should interpret this as: a. eye trauma. b. neurosurgical emergency. c. severe brainstem damage. d. indication of brain death.

ANS: B The sudden appearance of a fixed and dilated pupil(s) is a neurosurgical emergency. The nurse should immediately report this finding. Although a dilated pupil may be associated with eye trauma, this child has experienced a neurologic insult. Pinpoint pupils or bilateral fixed pupils for more than 5 minutes are indicative of brainstem damage. The unilateral fixed and dilated pupil is suggestive of damage on the same side of the brain. One fixed and dilated pupil is not suggestive of brain death.

10. Which is the usual presenting symptom for testicular cancer? a. Hard, painful mass b. Hard, painless mass c. Epididymis easily palpated d. Scrotal swelling and pain

ANS: B The usual presenting symptom for testicular cancer is a heavy, hard, painless mass that is either smooth or nodular and palpated on the testes. A hard, painful mass, an epididymis easily palpated, and scrotal swelling and pain are not the clinical presentations of testicular cancer.

2. Which strategies should the school nurse recommend implementing in the classroom for a child with attention deficit hyperactive disorder (ADHD)? (Select all that apply.) a. Schedule heavier subjects to be taught in the afternoon. b. Accompany verbal instructions by written format. c. Limit number of breaks taken during instructional periods. d. Allow more time for testing. e. Reduce homework and classroom assignments.

ANS: B, D, E Children with ADHD need an orderly, predictable, and consistent classroom environment with clear and consistent rules. Homework and classroom assignments may need to be reduced, and more time may need to be allotted for tests to allow the child to complete the task. Verbal instructions should be accompanied by visual references such as written instructions on the blackboard. Schedules may need to be arranged so that academic subjects are taught in the morning when the child is experiencing the effects of the morning dose of medication. Regular and frequent breaks in activity are helpful because sitting in one place for an extended time may be difficult.

6. The nurse is monitoring an infant for signs of increased intracranial pressure (ICP). Which are late signs of increased intracranial pressure (ICP) in an infant? (Select all that apply.) a. Tachycardia b. Alteration in pupil size and reactivity c. Increased motor response d. Extension or flexion posturing e. Cheyne-Stokes respirations

ANS: B, D, E Late signs of ICP in an infant or child include bradycardia, alteration in pupil size and reactivity, decreased motor response, extension or flexion posturing, and Cheyne-Stokes respirations.

4. A nurse is caring for a child who is near death. Which physical signs indicate the child is approaching death? (Select all that apply.) a. Body feels warm b. Tactile sensation decreasing c. Speech becomes rapid d. Change in respiratory pattern e. Difficulty swallowing

ANS: B, D, E Physical signs of approaching death include: tactile sensation beginning to decrease, a change in respiratory pattern, and difficulty swallowing. Even though there is a sensation of heat the body feels cool, not warm, and speech becomes slurred, not rapid.

8. Which neurologic diagnostic test gives a visualized horizontal and vertical cross-section of the brain at any axis? a. Nuclear brain scan b. Echoencephalography c. CT scan d. Magnetic resonance imaging (MRI)

ANS: C A CT scan provides a visualization of the horizontal and vertical cross-sections of the brain at any axis. A nuclear brain scan uses a radioisotope that accumulates where the blood-brain barrier is defective. Echoencephalography identifies shifts in midline structures of the brain as a result of intracranial lesions. MRI permits visualization of morphologic features of target structures and permits tissue discrimination that is unavailable with any other techniques.

26. When assessing the eyes of a neonate, the nurse observes opacity of the lens. This represents which impairment? a. Blindness b. Glaucoma c. Cataracts d. Retinoblastoma

ANS: C A cataract is opacity of the lens of the eye. The child may have visual impairment secondary to the cataract, but the opacity is a cataract. Glaucoma is increased intraocular pressure. Retinoblastoma is a tumor of the eye.

6. The nurse is discussing sexuality with the parents of an adolescent girl with moderate cognitive impairment. Which should the nurse consider when dealing with this issue? a. Sterilization is recommended for any adolescent with cognitive impairment. b. Sexual drive and interest are limited in individuals with cognitive impairment. c. Individuals with cognitive impairment need a well-defined, concrete code of sexual conduct. d. Sexual intercourse rarely occurs unless the individual with cognitive impairment is sexually abused.

ANS: C Adolescents with moderate cognitive impairment may be easily persuaded and lack judgment. A well-defined, concrete code of conduct with specific instructions for handling certain situations should be laid out for the adolescent. Permanent contraception by sterilization presents moral and ethical issues and may have psychological effects on the adolescent. It may be prohibited in some states. The adolescent needs to have practical sexual information regarding physical development and contraception. Cognitively impaired individuals may desire to marry and have families. The adolescent needs to be protected from individuals who may make intimate advances.

16. A 10-year-old boy on a bicycle has been hit by a car in front of the school. The school nurse immediately assesses airway, breathing, and circulation. The next nursing action: should be to a. place on side. b. take blood pressure. c. stabilize neck and spine. d. check scalp and back for bleeding.

ANS: C After determining that the child is breathing and has adequate circulation, the next action is to stabilize the neck and spine to prevent any additional trauma. The child's position should not be changed until the neck and spine are stabilized. Blood pressure is a later assessment. Less urgent, but an important assessment, is inspection of the scalp for bleeding.

23. A 3-year-old child is hospitalized after a submersion injury. The child's mother complains to the nurse, "Being at the hospital seems unnecessary when he is perfectly fine." The nurse's best reply should be: a. "He still needs a little extra oxygen." b. "I'm sure he is fine, but the doctor wants to make sure." c. "The reason for this is that complications could still occur." d. "It is important to observe for possible central nervous system problems."

ANS: C All children who have a submersion injury should be admitted to the hospital for observation. Although many children do not appear to have suffered adverse effects from the event, complications such as respiratory compromise and cerebral edema may occur 24 hours after the incident. The mother would not think the child is fine if oxygen were still required. The nurse should clarify that different complications can occur up to 24 hours later and that observations are necessary.

32. A nurse is preparing to perform a dressing change on a 6-year-old child with mild cognitive impairment (CI) who sustained a minor burn. Which strategy should the nurse use to prepare the child for this procedure? a. Verbally explain what will be done. b. Have the child watch a video on dressing changes. c. Demonstrate a dressing change on a doll. d. Explain the importance of keeping the burn area clean.

ANS: C Children with CI have a marked deficit in their ability to discriminate between two or more stimuli because of difficulty in recognizing the relevance of specific cues. However, these children can learn to discriminate if the cues are presented in an exaggerated, concrete form and if all extraneous stimuli are eliminated. Therefore, demonstration is preferable to verbal explanation, and learning should be directed toward mastering a skill rather than understanding the scientific principles underlying a procedure. Watching a video would require the use of both visual and auditory stimulation and might produce overload in the child with mild cognitive impairment. Explaining the importance of keeping the burn area clean would be too abstract for the child.

27. The mother of a 1-month-old infant tells the nurse she worries that her baby will get meningitis like her oldest son did when he was an infant. The nurse should base her response on which statement? a. Meningitis rarely occurs during infancy. b. Often a genetic predisposition to meningitis is found. c. Vaccination to prevent all types of meningitis is now available. d. Vaccination to prevent Haemophilus influenzae type B meningitis has decreased the frequency of this disease in children.

ANS: D H. influenzae type B meningitis has been virtually eradicated in areas of the world where the vaccine is administered routinely. Bacterial meningitis remains a serious illness in children. It is significant because of the residual damage caused by undiagnosed and untreated or inadequately treated cases. The leading causes of neonatal meningitis are the group B streptococci and Escherichia coli organisms. Meningitis is an extension of a variety of bacterial infections. No genetic predisposition exists. Vaccinations are not available for all of the potential causative organisms.

20. Which is considered a cardinal sign of diabetes mellitus? a. Nausea b. Seizures c. Impaired vision d. Frequent urination

ANS: D Hallmarks of diabetes mellitus are glycosuria, polyuria, and polydipsia. Nausea and seizures are not clinical manifestations of diabetes mellitus. Impaired vision is a long-term complication of the disease.

9. What is one of the major physical characteristics of the child with Down syndrome? a. Excessive height b. Spots on the palms c. Inflexibility of the joints d. Hypotonic musculature

ANS: D Hypotonic musculature is one of the major characteristics. Children with Down syndrome have short stature and a transverse palmar crease. Hyperflexibility is a characteristic of Down syndrome.

11. Which is characteristic of dishonest behavior in children ages 8 to 10 years? a. Cheating during games is now more common. b. Lying results from the inability to distinguish between fact and fantasy. c. They may steal because their sense of property rights is limited. d. They may lie to meet expectations set by others that they have been unable to attain.

ANS: D Older school-age children may lie to meet expectations set by others to which they have been unable to measure up. Cheating usually becomes less frequent as the child matures. In this age group, children are able to distinguish between fact and fantasy. Young children may lack a sense of property rights; older children may steal to supplement an inadequate allowance, or it may be an indication of serious problems.

13. Which clinical manifestation may occur in the child who is receiving too much methimazole (Tapazole) for the treatment of hyperthyroidism (Graves disease)? a. Seizures b. Enlargement of all lymph glands c. Pancreatitis or cholecystitis d. Lethargy and somnolence

ANS: D Parents should be aware of the signs of hypothyroidism that can occur from overdosage of the drug. The most common manifestations are lethargy and somnolence. Seizures and pancreatitis are not associated with the administration of Tapazole. Enlargement of the salivary and cervical lymph glands occurs.

7. Parents ask the nurse whether it is common for their school-age child to spend a lot of time with peers. The nurse should respond, explaining that the role of the peer group in the life of school-age children provides: a. opportunity to become defiant. b. time to remain dependent on their parents for a longer time. c. time to establish a one-on-one relationship with the opposite sex. d. security as they gain independence from their parents.

ANS: D Peer-group identification is an important factor in gaining independence from parents. Children learn how to relate to people in positions of leadership and authority and how to explore ideas and the physical environment. Becoming defiant in a peer-group relationship may lead to bullying. Peer-group identification helps in gaining independence rather than remaining dependent. One-on-one opposite sex relationships do not occur until adolescence. School-age children form peer groups of the same sex.

5. A child will start treatment for precocious puberty. The nurse recognizes that this will involve the injection of which synthetic medication? a. Thyrotropin b. Gonadotropins c. Somatotropic hormone d. Luteinizing hormone-releasing hormone

ANS: D Precocious puberty of central origin is treated with monthly subcutaneous injections of luteinizing hormone-releasing hormone. Thyrotropin, gonadotropins, and somatotropic hormone are not the appropriate therapies for precocious puberty.

13. An adolescent girl calls the nurse at the clinic because she had unprotected sex the night before and does not want to be pregnant. The nurse should explain that: a. it is too late to prevent an unwanted pregnancy. b. an abortion may be the best option if she is pregnant. c. Norplant can be administered to prevent pregnancy for up to 5 years. d. postcoital contraception is available to prevent implantation.

ANS: D Several emergency methods of contraception are available. Postcoital contraception options do exist. It is nontherapeutic to tell her it is too late or that an abortion is the best option. Norplant is not a postcoital contraceptive.

27. The nurse is caring for an 8-year-old child with type 1 diabetes. The nurse should teach the child to monitor for which manifestation of hypoglycemia? a. Lethargy b. Thirst c. Nausea and vomiting d. Shaky feeling and dizziness

ANS: D Some of the clinical manifestations of hypoglycemia include shaky feelings; dizziness; difficulty concentrating, speaking, focusing, or coordinating; sweating; and pallor. Lethargy, thirst, and nausea and vomiting are manifestations of hyperglycemia.

25. An adolescent teen has bulimia. Which assessment finding should the nurse expect to assess? a. Diarrhea b. Amenorrhea c. Cold intolerance d. Erosion of tooth enamel

ANS: D Some of the signs of bulimia include erosion of tooth enamel, increased dental caries from vomited gastric acid, throat complaints, fluid and electrolyte disturbances, and abdominal complaints from laxative abuse. Diarrhea is not a result of the vomiting. It may occur in patients with bulimia who also abuse laxatives. Amenorrhea and cold intolerance are characteristics of anorexia nervosa, which some bulimics have. These symptoms are related to the extreme low weight.

23. Which symptoms should the nurse expect to observe during the physical assessment of an adolescent girl with severe weight loss and disrupted metabolism associated with anorexia nervosa? a. Dysmenorrhea and oliguria b. Tachycardia and tachypnea c. Heat intolerance and increased blood pressure d. Lowered body temperature and brittle nails

ANS: D Symptoms of anorexia nervosa include lower body temperature, severe weight loss, decreased blood pressure, dry skin, brittle nails, altered metabolic activity, and presence of lanugo hair. Amenorrhea, rather than dysmenorrhea, and cold intolerance are manifestations of anorexia nervosa. Bradycardia, rather than tachycardia, may be present.

17. Nursing interventions to help the siblings of a child with special needs cope include: a. explaining to the siblings that embarrassment is unhealthy. b. encouraging the parents not to expect siblings to help them care for the child with special needs. c. providing information to the siblings about the child's condition only as they request it. d. suggesting to the parents ways of showing gratitude to the siblings who help care for the child with special needs.

ANS: D The presence of a child with special needs in a family will change the family dynamic. Siblings may be asked to take on additional responsibilities to help the parents to care for the child. The parents should show gratitude, such as an increase in allowance, special privileges, and verbal praise. Embarrassment may be associated with having a sibling with a chronic illness or disability. Parents must be able to respond in an appropriate manner without punishing the sibling. The parents may need assistance with the care of the child. Most siblings are positive about the extra responsibilities. The siblings need to be informed about the child's condition before a nonfamily member does so. The parents do not want the siblings to fantasize about what is wrong with the child.

15. Which statement is true about gonorrhea? a. It is caused by Treponema pallidum. b. Treatment is by multidose administration of penicillin. c. Treatment is by topical applications to lesions. d. Treatment of all sexual contacts is an essential part of treatment.

ANS: D The treatment plan should include finding and treating all sexual partners. Gonorrhea is caused by Neisseria gonorrhoeae. Syphilis is caused by T. pallidum. Primary treatment is with different antibiotics because of N. gonorrhoeae resistance to penicillin. Systemic therapy is necessary to treat this disease.

14. The nurse is talking with the parent of a child newly diagnosed with a chronic illness. The parent is upset and tearful. The nurse asks, "Whom do you talk to when something is worrying you?" This should be interpreted as: a. inappropriate, because the parent is so upset. b. a diversion of the present crisis to similar situations with which the parent has dealt. c. an intervention to find someone to help the parent. d. part of assessing the parent's available support system.

ANS: D These are important data for the nurse to obtain. This question will provide information about the marital relationship (whether the parent speaks to the spouse), alternate support systems, and ability to communicate. By assessing these areas, the nurse can facilitate the identification and use of community resources as needed. It is an important part of assessment information to determine the parent's support network. The nurse is obtaining information to help support the parent through the diagnosis. The parent is not in need of additional parenting help at this time.

14. The father of a 12-year-old child tells the nurse that he is concerned about his son getting "fat." His son is at the 50th percentile for height and the 75th percentile for weight on the growth chart. The most appropriate nursing action is to: a. reassure the father that his child is not fat. b. reassure the father that his child is just growing. c. suggest a low-calorie, low-fat diet. d. explain that this is typical of the growth pattern of boys at this age.

ANS: D This is a characteristic pattern of growth in preadolescent boys, where the growth in height has slowed in preparation for the pubertal growth spurt, but weight is still gained. The nurse should review this with both the father and the child and develop a plan to maintain physical exercise and a balanced diet. It is false reassurance to tell the father that his son is not fat. His weight is high for his height. The child needs to maintain his physical activity. The father is concerned, so an explanation is required. A nutritional diet with physical activity should be sufficient to maintain his balance.

21. Which term best describes a multidisciplinary approach to the management of a terminal illness that focuses on symptom control and support? a. Dying care b. Curative care c. Restorative care d. Palliative care

ANS: D This is one of the definitions of palliative care. The goal of palliative care is the achievement of the highest possible quality of life for patients and their families.

24. The parent of a child with diabetes mellitus asks the nurse when urine testing will be necessary. The nurse should explain that urine testing is necessary for which? a. Glucose is needed before administration of insulin. b. Glucose is needed four times a day. c. Glycosylated hemoglobin is required. d. Ketonuria is suspected.

ANS: D Urine testing is still performed to detect evidence of ketonuria. Urine testing for glucose is no longer indicated because of the poor correlation between blood glucose levels and glycosuria. Glycosylated hemoglobin analysis is performed on a blood sample.

18. A child is unconscious after a motor vehicle accident. The watery discharge from the nose tests positive for glucose. The nurse should recognize that this suggests: a. diabetic coma. b. brainstem injury. c. upper respiratory tract infection. d. leaking of cerebrospinal fluid (CSF).

ANS: D Watery discharge from the nose that is positive for glucose suggests leaking of CSF from a skull fracture and is not associated with diabetes or respiratory tract infection. The fluid is probably CSF from a skull fracture and does not signify whether the brainstem is involved.

22. The weight loss of anorexia nervosa is usually triggered by a. sexual abuse. b. school failure. c. independence from family. d. traumatic interpersonal conflict.

ANS: D Weight loss may be triggered by a typical adolescent crisis such as the onset of menstruation or a traumatic interpersonal incident; situations of severe family stress, such as parental separation or divorce; or circumstances in which the young person lacks personal control, such as being teased, changing schools, or entering college. "Sexual abuse," "school failure," and "independence from family" are not part of the behavioral characteristics of anorexia nervosa.

16. The school nurse has been asked to begin teaching sex education in the fifth grade. The nurse should recognize that: a. children in fifth grade are too young for sex education. b. children should be discouraged from asking too many questions. c. correct terminology should be reserved for children who are older. d. sex can be presented as a normal part of growth and development.

ANS: D When sexual information is presented to school-age children, sex should be treated as a normal part of growth and development. Fifth-graders are usually 10 or 11 years old. This age is not too young to speak about physiologic changes in their bodies. They should be encouraged to ask questions. Preadolescents need precise and concrete information.

2. A nurse teaches parents that team play is important for school-age children. Which can children develop by experiencing team play? (Select all that apply.) a. Achieve personal goals over group goals. b. Learn complex rules. c. Experience competition. d. Learn about division of labor.

BCD Team play helps stimulate cognitive growth because children are called on to learn many complex rules, make judgments about those rules, plan strategies, and assess the strengths and weaknesses of members of their own team and members of the opposing team. Team play can also contribute to children's social, intellectual, and skill growth. Children work hard to develop the skills needed to become team members, to improve their contribution to the group, and to anticipate the consequences of their behavior for the group. Team play teaches children to modify or exchange personal goals for goals of the group; it also teaches them that division of labor is an effective strategy for attaining a goal.


Kaugnay na mga set ng pag-aaral

The United States enters the War 3

View Set

Quiz: Module 06 Wireless Networking

View Set

Supervision and Human Relations - Exam 2 Study Set

View Set

Exam 2 TX GOVT (Ch 6, Ch 5, Ch 4)

View Set

Pt 8, Pt 7, pt 6, Pt 5, Pt 4, Pt 3, Pt 2, Pt 1, Test 1

View Set

Canvas Final Exam Review Notes 3

View Set

NURS 1118 4.1 Outcomes Patient Education

View Set

Penguin Dictionary of Literary Terms and Criticism - A II

View Set